Vous êtes sur la page 1sur 61

Aortic Aneurysm

Adrenal adenoma

A 48 year-old male patient with refractory hypertension on four anti-


hypertensive medications is found to have Cushings syndrome.
Review of his medications reveals no exogenous glucocorticoid
administration. Which lab result would confirm that this patient is a
candidate for laparoscopic adrenalectomy?

Suppression of cortisol with low-dose dexamethasone


High Cortisol
Low ACTH
Mild increase of cortisol in response to administration of corticotrophin
releasing hormone
High ACTH

Discussion: Work-up of the patient with Cushings syndrome requires


identification of the source of the excess cortisol production. One would
expect Cushings syndrome of any etiology to have an elevated cortisol
level. In a normal patient, cortisol levels would be suppressed by
administration of low-dose dexamethasone. A high ACTH is indicative
of a pituitary source, also known as Cushings disease. A low ACTH
level is indicative of a non-pituitary source; in this case making an
adrenal source the most likely etiology.

What structure does the left adrenal vein empty into?

Left renal vein


Left common iliac vein
Left phrenic vein
Inferior vena cava
Right renal vein

Your answer is correct


Discussion: The shorter distance between the right adrenal gland and
inferior vena cava allows for a direct communication between the right
adrenal vein and the inferior vena cava. This shorter distance can
make control of the right adrenal vein technically challenging. The left
adrenal vein, however, is longer and emptied directly into the left renal
vein. Both adrenal glands have arterial blood supply via direct
branches from the aorta, renal arteries, and phrenic arteries.

A 24 year-old asymptomatic male was found to have a 2.5cm left


adrenal mass on CT scan of the abdomen for abdominal pain. He was
also found to have appendicitis and underwent laparascopic
appendectomy uneventfully. Upon discharge, you explain to the patient
that he has a small adrenal nodule and he is given a follow up
appointment. Wish test would NOT be indicated to assess if this mass
is a functional adenoma?

Dexamethasone suppression test


Plasma Testosterone Level
Urinary and serum metanephrine/normetanephrine levels
Renin/Aldosterone level
Serum electrolyte level

According to the NIH consensus on management of asymptomatic


adrenal mass, the patient should be tested for renin/aldosterone level
and serum electrolyte levels to rule out aldostrenoma; dexamethasone
suppression test to rule out Cushing Sydrome and sub-clinical Cushing
syndrome, and Urinary and serum metanephrine/normetanephrine
levels to rule out Pheochromocytoma. Sex hormone levels should be
ordered when virilization or feminization is apparent on patient
presentation.

48yearoldmalesmokerwith30packyearhistorywithnoadditionalsignificantpast
medicalhistoryisbroughttotheemergencyroomafterafallfromscaffoldingat
work.Onarrival,hewasnormotensiveat110/60,P:80,withtemperatureof37.On
physicalexam,hecomplainedoflegpainontheleftandtheremainderofhisphysical
examincludingneurologicalevaluationwasnormal.HisPAandlateralchestxray
revealednoevidenceofpneumothorax.HeunderwentCTscanoftheabdomenand
pelviswhichrevealeda6.2cmleftadrenalnodule.Laboratoryresultsdidnot
demonstratefunctionality.Whatisthenextstepinmanagement?

Openadrenalectomy
RuleoutlungcancerbyobtainingCTscan
Observation
Laparascopicadrenalectomy
CTguidedbiopsyofadrenalmass

ThisisapatientwithhistoryofsmokingwithanincidentallyfoundadrenalmassonCT
Scan.Theriskofcancerincreaseswithincreasingsize.Atthispointwithlesionmore
than6cm,itshouldberecommendedthatheundergolaparoscopicadrenalectomy.
Althoughnotclearlylinked,thereisanassociationbetweensmokinganddevelopmentof
adrenalmalignancy.

A 42 year-old male undergoes a CT scan of the chest, abdomen, and


pelvis following a motor vehicle crash and is found to have a 3cm left
adrenal nodule. What is the most appropriate next step in
management?

basic metabolic profile and 24 hour urine samples


Laparoscopic left adrenalectomy
No further work-up is indicated
Repeat CT scan in 3 months
CT guided core needle biopsy

Discussion: Work-up of the adrenal incidentaloma is an important part


of any physicians knowledge base. Any adrenal nodule first requires
evaluation of whether it is functional or non-functional. If it is non-
functional, evaluation for any potential malignancy producing an
adrenal metastasis is warranted. For non-functional adrenal nodules
less than 4cm, interval follow-up via CT scan to monitor for growth of
the nodule is indicated. Functional nodules, or non-functional nodules
4cm or larger should be surgically removed. CT guided core biopsy
should only be reserved to document a metastatic lesion to the adrenal
gland after the primary malignancy has been identified AND a
pheochromocytoma has been ruled out.

A 62 year old male underwent a CT scan for evaluation for right lower
quadrant pain to rule out appendicitis. His medical history is
significant for hypertension, hypercholesterolemia, and recent MI three
months ago that required cardiac catheterization and placement of
drug eluting stent. His CT scan was unremarkable for appendicitis but
incidentally he was found to have 2.5 cm adrenal nodule on the right
side. Further work up revealed normal serum and urine catecholamine
levels, and a plasma aldosterone/renin ratio of 31. His current
medication includes a beta-blocker, ACE-inhibitor, and calcium channel
blocker. On physical exam, he was normotensive at 120/80, P: 65 and
otherwise in good health. What would you advise this patient at this
point?

Observe the mass as it is a small incidentaloma that is hormonally


inactive and the there is no benefit to excision
Open adrenalectomy for functional mass
Observe the mass as the risk outweighs the benefit at this point
Laparascopic adrenalectomy for non-functional adrenal mass
Laparascopic adrenalectomy for functional adrenal mass

This is a functional aldosteronoma with plasma aldosterone/renin ratio


over 30. In a normal circumstance, where there is a proven functional
adrenal nodule, the patient would undergo adrelenalectomy. In this
patient with recent MI with drug eluting stent, he will need to be on
anticoagulants for more than three months. Furthermore, his blood
pressure is currently is well controlled with his anti-hypertensive
medications. The risk of this patient undergoing the operation
outweighs the benefit at this point. He will be better served with
observation until he is medically optimized or he becomes more
symptomatic.
A38yearoldfemalepresentswithfrequentboutsofheadaches.Shehasbeenonthreedifferentanti
hypertensivemedicationssincetheageof36.Despitemaximizingthedosageoftheantihypertensive
medications,hersystolicbloodpressurecontinuestoremaininthe160s.SheiscurrentlytakinganACE
Inhibitor,aBBlocker,andaCalciumchannelblocker.ShehadCTscanwhenshewas30yearsoldafter
anautomobileaccidentandwastoldthatshemayhavesmallnodularmassonherrightadrenalgland.

Onphysicalexam,sheiswellappearingwithnopalpablemassonabdominalexam.Hertemperatureis
98.7,Bloodpressureis160/60,HRof80.Herfundoscopicexaminationisunremarkable.Routineblood
testsshowedthefollowing:

BMP

Sodium144meq/L
Sodium144meq/L
Pottasium2.9meq/L
Chloride103meq/L
Bicarbonate30meq/L
Creatinine1.1mg/dL
UrinalysisNormal
UrineSodiumlow
UrinePotassiumhigh
Whatisthenextbeststepinmanagementforthispatient?
CTscanoftheabdomenandpelvis
Measurementofplasmametanephrineandnormetanephrinelevels
Dexamethasonesuppressiontest
RenalAngiography

Plasmarenin/aldosteronelevel
Youranswerisnotcorrect

Thispatienthasanaldosteronoma.Thedifferentialdiagnosisforpatientwhopresentswithuncontrolled
hypertensionincludesPheochromocytoma,Aldosteronoma,CushingsDisease,andRenalHypertension.
Thepatientabovepresentedwithuncontrolledhypertensionaccompaniedbylowserumpotassiumlevel
andslightlyelevatedserumsodiumlevel.Furthermore,thispatientalsohaselevatedurinepotassiumwhich
isconsistentwiththediagnosisofaldosteronoma.

A55yearoldmanisevaluatedfora10monthhistoryofrecurrentepisodesofsweating,palpitations,and
headaches.Heotherwisehasanunremarkablemedicalhistory.HeiscurrentlytakingAtenololand
Lisinopriltocontrolhisbloodpressure.Physicalexamrevealedaveryanxiousappearingmanwithsweaty
palmsandneckexamsignificantfornormalthyroidglands.Hisvitalsignsincludeatemperatureof37.8C,
bloodpressureof162/95,pulserateof90bpmandrespiratoryrateof16breathsperminute.

Laboratorystudiesrevealedelevatedspotplasmametanephrinelevels.HesubsequentlyundergoesCTscan
oftheabdomenandpelvisdemonstratesa5mmnodulewithintherightadrenal.MRIoftheabdomen
revealsasimilarfinding,otherwisenegative.Whatisthenextbeststepinthemanagementofthispatient?

PETScan

LaparoscopicrightAdrenalectomy
VenographywithvenoussamplingbilateralAdrenals
MIBGScan
Ultrasoundoftheabdomen
Youranswerisnotcorrect

Thispatienthassignsandsymptomsofpheochromocytoma.Biochemicallyitwasshownthathehas
elevatedmetanephrinelevelsconsistentwithdiagnosisofpheochromocytoma.However,neithertheCT
scanoftheabdomennortheMRIoftheabdomencouldbeconsidereddefinitivelocalizationasthe?mass
seenisverysmallandcouldbeanincidentaladenomaorevenafalsepositiveread(sincesosmall).The
nexttesttolookforadrenalmassistheMIBGscan.Thisisatestthatutilizesaradioactiveisotopethatis
selectivelytakenupbytissuesthatsecretecatecholamines.MIBGscanisusuallyreservedforpatients
wheretheCTscanfindingisequivocalortodiagnoseextraadrenalpheochromocytoma.Inthiscaseit
wouldnotbehelpfulasevenifthis5mmmassisthelesion,itistoosmalltopickupbynuclearscanand
alsoitiswithintheadrenal,sonothelpfulasadrenalswillhaveanormalphysiologicuptakebyMIBG
(thatiswhyitishelpfultoassessanextraadrenalmass).Venographywithvenoussamplingofbilateral
adrenalglandsmaybeusefulwhendealingwithbilateraladrenalmassesandforplanningsurgical
resectiontoseewhetheranoduleisbiochemicallyactive.Thedownsidetothetestisthatitisinvasiveand
shouldbereservedafternoninvasivetestinghasbeenattempted.
A22yearoldfemalewithclinicalsuspicionforhypercortisolismisseenbytheendocrinologistforfollow
upafterobtaininglaboratorywork.InadditiontoherCushinoidfeatures,allofthelabresultsareconsistent
withhypercortisolism.Sheisnowinyourofficetodiscusstheresultsandthenextstepin
management.Herlaboratoryresultsareasfollows.:

Adrenocorticotropichormone:low
Urinefreecortisol:Elevated
Cortisol(8AM)
After1mgofdexamethasonethenightbefore:Elevated
After8mgofdexamethasonethenightbefore:Elevated

Whattestdoyouordernext?
Serum/urine catecholamine
CT scan of the head

CT Scan of the abdomen


MRI of the head
Cosyntropin stimulation test
Youransweriscorrect

This is a patient with suspected hypercortisolism. With low ACTH and cortisol levels that are not
suppressed after high and low dose dexamethasone suppression test, one of the suspicions is
that the patient has an ectopic adrenal nodule that is responsible for the results shown above. On
the contrary, patients who have pituitary adenomas resulting in Cushings will have elevated
ACTH, elevated urine free cortisol, and high dexamethasone suppression test will partially
depress the cortisol level. Thus, in this patient, one of the areas to evaluate is the adrenal gland
by obtaining CT Scan of the abdomen.
Anorectal Disease

1) A38yearoldwomancomestoyourofficebecauseofa4weekhistoryofpainfulbowelmovements
associatedwithasmallamountofbloodonthetoilettissuewithwiping.Shehasnohistoryofsimilar
episodesbutisveryconcernedbecausehergrandfatherdiedofcoloncancer.Shehasoccasional
constipationbuthasotherwisebeenhealthy.Thepatientisinamonogamousrelationshipwithherhusband.
Withtheexceptionoftopicalhemorrhoidointment,shetakesnomedications.

Onphysicalexamination,hertemperatureis98.6F(37.0C),pulseis80/minandregular,respirationsare
14/min,andbloodpressureis110/70mmHg.Physicalexaminationshowsaposteriormidlineskintagthat
istendertopalpation.Digitalrectalexaminationcannotbecompletedbecauseofseverepainandsphincter
spasm.

Whichofthefollowingisthemostlikelycauseofthispatientssymptoms?

Squamouscarcinomaofanus

Chronicanalfissure
Thrombosedhemorrhoid
Condylomaaccuminata
Perianalabscess
Youransweriscorrect
Thispatienthasaclassichistoryforachronicanalfissure.Minoranaltraumafromconstipation(usual
history)ordiarrheacanleadtoanacuteanalfissure.Thepatientmaynothavepaininitiallybutmaysee
brightredbloodonthetoilettissue.Painmayoccurwithrepeatedepisode.Initiallythepainiswith
defecation.Withtimethepainmaybeinitiatedbydefecationbutlingerslongafterwardduetosphincter
spasmwhichleadstoischemiaoftheanodermwhichinturnsleadstoachroniculcerandscarringintothe
internalanalsphincter.Patientsavoiddefecatingduetothepainleadingtomoreconstipationand
propagationoftheproblem.Chronicanalfissuresareusuallylocatedintheposteriormidline,theleast
distensibleareaofthesphinctermechanism.Painandsphincterspasmmakesitverydifficulttodoadigital
rectalexamwithoutanesthesia.Thereisoftenaprominentskintagorsentinelpilethatisdistaltothe
fissurethatliesintheanalcanal.Thismayappearedematousandbepainfulwhenpalpated,andisoften
confusedasathrombosedhemorrhoidoranalwarts(condylomaaccuminata)byinexperiencedclinicians.
Carefullyspreadingtheanalskinandsphincterwillrevealthesmallulcerproximaltothetag,butthiscan
bedifficulttoaccomplishwithoutanesthesiainmanypatients.

Thrombosedhemorrhoidshaveanacuteonsetandthepatientcanusuallyfeelanoduleadjacenttotheanal
canal.Usuallyoneseesablueblackcolorednodulethatistendertotouch.Thereusuallyisnoevidenceof
sphincterspasmorantecedenthistoryofbloodonthetissue.

Aperianalabscesswillalsocausepain,andthepatientoftencanfeelatenderlumpinthearea.Defecation
maybepainfulwithanabscess,sincetheseusuallystartasananalcryptabscessandcancommunicate
throughaportionofthesphinctertothesubcutaneousperianaltissue.Thesemayappearasafluctuantarea
witherythematousoverlyingskinorasjustatenderinduratedarea.Intramuscularabscessesshowno
perianalskinchanges,buttheyusuallyareassociatedwithverypainfuldigitalrectalexams.

CondylomaaccuminataoranalwartsduetoHPVinfectionappearassolitaryormultiplepapulesthat
sometimescanbequitelargeandbulky.Theyareusuallyassociatedwithlessseverepainbutbulkywarts
canbecomeverypainful.Analwartsareassociatedwithariskfordevelopingsquamouscellcarcinoma.

Squamouscellcarcinomacanoccurinandaroundtheanalcanal.Theseareoftenpainfulandcanbe
confusedwithchronicfissures.Analcancerscanoccuranywherearoundtheanusandachroniculcerthat
isnotintheposteriormidlineshouldalwaysbesuspiciousforcancer.Chronicposteriormidlinefissures
thatdonotrespondtotreatmentshouldbebiopsiedtoruleoutmalignancy.

2) Apreviouslyhealthy27yearoldmancomestotheemergencydepartmentbecauseofperianalpain,
swelling,anddrainageofasmallamountofpusforthepast2weeks.Hereportsa10pound(4.5kg)
weightloss,abdominalcramping,andintermittentmucusinhisstools.Hereportshaving7to8stoolsper
day.Hisonlymedicationisloperamide.

Onphysicalexamination,histemperatureis99.0F(37.2C),pulseis88/minandregular,respirationsare
14/min,andbloodpressureis120/80mmHg.Theabdomenisflatandsoftwithmildtendernessto
palpationovertherightlowerquadrant.Rectalexaminationshowsachronicappearingfistulainano.

Whichofthefollowingisthemostlikelydiagnosis?

Colorectalcancer
Squamouscarcinomaofanus
Irritableboweldisease

Crohn'sdisease
Ulcerativecolitis
Youransweriscorrect
ThispatienthasaclassichistoryforCrohnsdisease,oneofthechronicinflammatoryboweldiseases.
Chronicdiarrhea,crampingandmucusinthestoolisafindingoftenassociatedwitheitherCrohnsdisease
orulcerativecolitis.Ulcerativecolitis(UC),ischaracterizedbymucosabasedinflammationandalways
involvestherectumwithvaryingdegreesofproximalextensioninthecolon.Perianaldiseaseandfistulas
areusuallynotassociatedwithUC.

CrohnsdiseaseischaracterizedbyfullthicknessinflammationintheGItract.Therecanbeareasof
normaltissuebetweenareasofinflammation(skipdisease)andrectalsparingiscommon.Perianaldisease,
(includingabscesses,fissuresandfistulae)iscommonwithCrohnsdisease,andsomepatientsdevelop
severediseasewithmultiplefistulae(wateringcananus).

Patientswithcolorectalcanceroftenareasymptomatic.Grossrectalbleedingwithoutdiarrheacanbeasign
ofadistalcolorectalcancer.Obstructivesymptomsaremorecommon,andcanbeassociatedwith
paradoxicaldiarrhea,butthisisusuallylesssevereandnotassociatedwithsignificantamountsofmucous.
Perianaldiseaseisusuallynotassociatedwithcolorectalcancer.Patientswithcolorectalcancerareusually
olderunlessthereisageneticrisksuchaswithfamilialpolyposis.

Perianalsquamouscellcancerscanoccurinyoungindividuals,particularlythosewithahistoryofHCV
infections.Theothersymptomsmanifestedinthispatientarerarelypresent.

Diarrheacanbeaprominentsymptominsomepatientswithirritablebowelsyndrome(IBS),butperianal
diseaseisnotamanifestationofIBS.

3)A28yearoldwomancomestoyourofficewithcomplaintsofhemorrhoids.Shestatesthatshehas
hadintermittentbleedingforthepast3months,andshereportssomethingisfallingoutofmyrectumwith
everybowelmovementandIhavetopushitbackin.Hersymptomshavenotrespondedtotopical
hemorrhoidointments.Thepatienthas3childrenbornbythevaginalroutewithoutcomplications.Her
medicalhistoryisotherwiseunremarkable.

Onphysicalexamination,hertemperatureis98.6F(37.0C),pulseis60/minandregular,respirationsare
12/min,andbloodpressureis100/60mmHg.Physicalexaminationshowsrectalmucosathatprotrudes
fromtheanusintheleftlateralpositionwhenshestrains.

Whichofthefollowingisthemostlikelydiagnosis?

Rectalprolapse
Externalhemorrhoids

Internalhemorrhoids
Hypertrophiedanalpapilla
Rectalpolyp
Youransweriscorrect

Thisyoungwomanpresentswithprolapsinginternalhemorrhoids.Theintermittentbrightredbleeding
associatedwithprolapsingtissueisacommonhistory.Hemorrhoidsarecommonlyassociatedwitha
historyofpregnancyandstrainingatdefecation.Thereisusuallyminimaltonopaininvolvedsincethe
prolapsinginternalhemorrhoidsarewithintherectalmucosaandreceiveentericinnervation.Thedilated
hemorrhoidsandoverlyingrectalmucosaaresusceptibletobleedingfromstraininganddefecation.
Externalhemorrhoidsarenotreducible.Theyarecoveredwithsquamousmucosaandarelocatedunder
theanodermintheanalcanal.Anodermhassomaticinnervation,andthrombosisandinflammationof
externalhemorhhoidscausesmoderatetoseverepain.Bleedingfromthrombosedhemorrhoidsoccurswhen
thecloterodesthroughtheskin.Externalhemorrhoidsthatarenotthrombosedarenotlikelytobleed.

Prolapsedinternalhemorrhoidsconsistofthesubmucosaltissuewiththedilatedhemorrhoidsandthe
overlyingrectalmucosa.Therearethreemaincolumnsofhemorrhoidsrightanterior,rightlateralandleft
posterior.Oneorallthreeofthecolumnsmaybeenlargedandprolapse.

Rectalprolapsereferstofullthickness(mucosa,submucosaandmuscularis)prolapseoftherectumthrough
theanus.Thispresentsasaconcentriccone,andasmallsegmentofrectumcanprolapseaswellasvery
largesegments.

Rarely,apedunculatedrectalpolypcanprolapseandcausesimilarsymptomstoaprolapsedhemorrhoid,
butiseasilydistinguishedonphysicalexamincludinganoscopy.

Hypertrophiedanalpapillaeoccuratthedentatelineandareusuallycausedfromanalinflammation,
crytotitsandpapillitis.Smallhypertrophiedpapillaearecommonandassociatedwithhemorrhoiddisease.
Occasionallyonecangetquitlargeandprolapse.Theyappearasafirm,tannishwhitetissue,andareeasily
identifiedbypalpationordirectvisualization

4)Apreviouslyhealthy21yearoldmancomestotheemergencydepartmentwithsevereanalpainthat
began24hoursagoandhasworsenedduringthepast3hours.Hetakesnomedications.

Onphysicalexamination,histemperatureis99.0F(37.2C),pulseis105/minandregular,respirationsare
16/min,andbloodpressureis120/80mmHg.Examinationoftheperineumshowsnoabnormalities.Rectal
examinationcannotbecompletedbecauseofseverepain.

Whichofthefollowingisthebestnextstepinmanagement?
Examinationunderanesthesiainoperatingroom
Antibioticsfor7days
Applicationoftopicalxylocainejelly
Computedtomographyofthepelvis
Endorectalultrasound
Youransweriscorrect

Acuteanalpainshouldalwaysinitiateanexaminationandnotbetreatedoverthetelephone.Thispatients
historyandphysicalfindingsarehighlysuspiciousforanintramuscularperirectalabscess.Athorough
examinationisnecessary,andbecauseofpainshouldbeperformedurgentlyunderanesthesiainthe
operatingroomwiththepatientconsentedtodrainanabscessordoothermeasurestoobtainthediagnosis
andtreattheproblem.

Empirictherapywithantibioticsaloneisnotappropriateforaperirectalabscesswhichalsoneedstobe
drained.Anurgentexaminationisnecessarytoconfirmthediagnosis.

Computedtomography(CT)scansshouldbedoneonlytoinvestigateaproblemidentifiedonacomplete
examination,andshouldnotbedonefirsttoruleoutdisease.

Endoscopicultrasonographyshouldneverbedonebeforeacompleteanorectalexamination.
Topicalanestheticssuchaslidocaine(Xylocaine)jellyareusuallyinadequatetocontrolpaininordertodo
acompleteanorectalexamination.Treatingthepainwithtopicalanestheticswithoutdeterminingthecause
ofthepainisnotappropriate.

5)A40yearoldmancomestotheemergencydepartmentbecauseofa10dayhistoryofperianalpain
withdischarge.Hismedicalhistoryisunremarkable,andhetakesnomedications.

Onphysicalexamination,histemperatureis99.5F(37.5C),pulseis90/minandregular,respirationsare
14/min,andbloodpressureis130/80mmHg.Examinationshowsa2cmholefromtheanalvergeinthe
leftposteriorlateralposition;theopeningisdrainingasmallamountofpus.Thesurroundingtissueis
mildlytender.

Perioperativeexaminationoftheanalcanalismostlikelytorevealaninternalopeningtoafistulaatwhich
ofthefollowinglocations?
Leftanteriorrectum
Leftposteriordentateline
Anteriormiddentateline

Leftposteriorrectum
Posteriormiddentateline
Youranswerisnotcorrect

Thismanhasafistulainanooranalfistula,afistulabetweentheanorectaljunctionandtheskin.These
fistulausuallyariseafterdrainageofaperianalorperirectalabscessandrepresentatubeofchronic
granulationtissuesthatfailstohealafterdrainageoftheabscess.Perirectalabscessesusuallystartinthe
analcryptsanderodethroughthesphinctermuscle.Aperianalabscessoccursiftheinfectionerodestothe
perianalskinthroughthesubcutaneousportionofthesphincter.Aperirectalabscessoccurswhenthe
abscesserodesthroughthedeeperportionsofthesphincterintotheperirectalandischiorectalspaces.
Sometimestheabscessesdrainspontaneouslybutoftenrequireincisionanddrainage.Anypatientwitha
perianalorperirectalabscessshouldbeadvisedthatafistulaisverylikelyafterdrainage,andthiscanoccur
evenmonthslaterafterpresumedhealing.Theopeningontheperianalorperirectalskinisthesecondary
opening.Theprimaryopeningisatthedentatelinewheretheinitiatingcryptoglandularabscessoccurred.

ThepaththatthefistulatakesispredictableaccordingtoGoodsallsrule.Whenthesecondaryopening
(external)isanteriortothemidtransverseplaneoftheanus,thefistulafollowsaradialcoursetotheanus
andtheprimaryopeningatthedentateline.Ifthesecondaryopeningisposteriortothetransverseplane,
thefistulawillfollowacoursebacktotheposteriormidlinedentateline.Themechanismforthisisnot
clearbutmayberelatedtofailureoffusionofthelongitudinalmuscleandtheexternalsphincterinthe
posteriormidline.Forsecondaryopeningsmorethan2cmfromtheanalverge,Goodsallsrulemaynot
applyasevenanterioropeningsmaytrackbacktotheposteriormidlineanus.Goodsallsruledoesnot
applytopatientswithCrohnsdisease.

6)Threemonthsfollowingdrainageofaperianalabscess,a33yearoldmancomestoseethephysician
becauseofintermittentperianalpainanddrainage.Henoticesanoccasionaldropofblood.Thedrainage
doesnotseemrelatedtothetimingofdefecation.Hispostoperativecoursehadbeenuneventfuluntil
approximately3weeksago.

Onphysicalexamination,histemperatureis98.6F(37.0C),pulseis70/minandregular,respirationsare
14/min,andbloodpressureis120/70mmHg.

Whichofthefollowingwillmostlikelybediscoveredonrectalexamination?
Analfissure
Internalhemorrhoids
Externalhemorrhoids

Fistulainano
Recurrentabscess
Youransweriscorrect

Thismanhasafistulainanooranalfistula,afistulabetweentheanorectaljunctionandtheskin.These
fistulausuallyariseafterdrainageofaperianalorperirectalabscessandrepresentatubeofchronic
granulationtissuesthatfailstohealafterdrainageoftheabscess.Perianalandperirectalabscessesusually
startintheanalcryptsanderodethroughthesphinctermuscle.Aperianalabscessoccursiftheinfection
erodestotheperianalskinthroughthesubcutaneousportionofthesphincter.Aperirectalabscessoccurs
whentheabscesserodesthroughthedeeperportionsofthesphincterintotheperirectalandischiorectal
spaces.Sometimestheabscessesdrainspontaneouslybutoftenrequireincisionanddrainage.Anypatient
withaperianalorperirectalabscessshouldbeadvisedthatafistulaisverylikelyafterdrainage,andthis
canoccurevenmonthslaterafterpresumedhealing.Theopeningontheperianalorperirectalskinisthe
secondaryopening.Theprimaryopeningisatthedentatelinewheretheinitiatingcryptoglandularabscess
occurred.

Arecurrentabscessusuallyoccursforthesamereasonasafistula,however,thesecondaryopening(skin)
hashealedandspontaneousdrainagehasnotoccurred.

Internalhemorrhoidsconsistofsubmucosaltissuewithdilatedhemorrhoidsandoverlyingrectalmucosa.
Therearethreemaincolumnsofhemorrhoidsrightanterior,rightlateralandleftposterior.Oneorall
threeofthecolumnsmaybeenlargedandprolapse.Thehemorrhoidsareoccasionallyassociatedwith
painlessdischargeofmucousandintermittentbleedoftenwithdefecation.

Externalhemorrhoidsarecoveredwithsquamousmucosaandarelocatedundertheanodermintheanal
canal.Anodermhassomaticinnervation,andthrombosisandinflammationofexternalhemorhhoidscauses
moderatetoseverepain.Bleedingfromthrombosedhemorrhoidsoccurswhenthecloterodesthroughthe
skin.

Minoranaltraumafromconstipation(usualhistory)ordiarrheacanleadtoanacuteanalfissure.The
patientmaynothavepaininitiallybutmayseebrightredbloodonthetoilettissue.Painmayoccurwith
repeatedepisode.Initiallythepainiswithdefecation.Withtimethepainmaybeinitiatedbydefecationbut
lingerslongafterwardduetosphincterspasmwhichleadstoischemiaoftheanodermwhichinturnsleads
toachronicanalfissurewhichismanifestedasasmallulcerandscarringintotheinternalanalsphincter.
Patientsavoiddefecatingduetothepainleadingtomoreconstipationandpropagationoftheproblem.
Chronicanalfissuresareusuallylocatedintheposteriormidline,theleastdistensibleareaofthesphincter
mechanism.Painandsphincterspasmmakesitverydifficulttodoadigitalrectalexamwithoutanesthesia.
Thereisoftenaprominentskintagorsentinelpilethatisdistaltothefissurethatliesintheanalcanal.This
mayappearedematousandbepainfulwhenpalpated,andisoftenconfusedasathrombosedhemorrhoidor
analwarts(condylomaaccuminata)byinexperiencedclinicians.Carefullyspreadingtheanalskinand
sphincterwillrevealthesmallulcerproximaltothetag,butthiscanbedifficulttoaccomplishwithout
anesthesiainmanypatients.

Appendictis

1)Pathophysiologyofappendicitis:

A.Obstructionofthelumenbylymphoidhyperplasiaorfecalith
B.Continuedmucousproductionwithdistentionoftheappendix
C.Venousobstruction
D.Arterialobstruction
E.Ischemia
F.Gangrene
G.Perforation
2)Appendicitisisthoughttobecausedby:

obstructionoftheappendiceallumen

idiopathicinflammation

subclinicaltrauma

amucoceletumor

abacterialinfection

Explanation:

Thepathophysiologyofappendicitisisnotuniformacrossages.Thus,whileappendicitiscanoccur(much
lesscommonly)duetoischemic,infectious,oroncologicprocess,theoverallprevailingmechanismis
lumenalobstructionthatcausesproximalappendicealinflammationandsubsequentrupture.

3)Whichofthefollowingsymptomsismostindicativeofacuteappendicitisinapatientwithrightlower
quadrantpain?

Priorepisodesofabdominalpain

Rigorsandhighfever

Bloodydiarrhea

Anorexia

Painprecedingvomiting

Explanation:

Appendicitisstartswithobstructionoftheappendixwhichcausespain,andprogressestoileuswhichleads
tothesymptomsofnauseaandvomiting.Rigorsandhighfeverwouldbesignsofadvancedappendicitis
duetoperforationandperitonitis.Priorepisodesofabdominalpainarerarewithappendicitis.Anorexiais
almostalwaysassociatedwithappendicitis,butisnotspecific.Bloodystoolsarenotseenwithappendicitis
butcanbeseenwithMeckel'sdiverticulumduetoheterotopicgastricmucosaandacidproductionleading
toulceration.

4)Rovsing'ssignissaidtobepositivewhenthepatientfeelspainintherightlowerquadrantwithwhich
ofthefollowingmaneuvers?

passiveextensionoftherighthip

passivestretchoftherighthip

activeflexionoftherighthip

internalrotationoftherighthip

deeppalpationintheleftlowerquadrant

Explanation:Rovsing'ssigniselicitedwhenpressureappliedintheleftlowerquadrantproducespainto
therightlowerquadrant.
Thepsoassigniselicitedbystretchoractiveflexionoftherighthip;thepatientwithacuteappendicitis
andaretrocecalappendixwilltypicallyhavepainwiththesemaneuversastheinflamedorganliesuponthe
rightiliopsoasmuscle.
Theobturatorsigniselicitedwiththepatientinthesupineposition,withpassiverotationoftheflexed
rightthigh;painwiththismaneuversuggestsapelviclocationoftheacutelyinflamedappendix.
Alloftheseareconsideredperitonealsignsandshouldbesoughtintheexaminationofapatientwithacute
appendicitis.Notethat,dependingupontheanatomiclocationoftheappendix,notallsignsmaybepresent.

5)An18yearoldmanpresentstotheEmergencyDepartmentwitha14hourhistoryofabdominalpain
whichhasnowlocalizedtotherightlowerquadrant.Onphysicalexamination,thereistendernessofdeep
palpationintherightlowerquadrant,withoutguardingorrebound.TheRovsing'ssignisnegative.Which
ofthefollowingadditionalphysicalfindingswouldbemostconsistentwithadiagnosisofappendicitisif
positive?

Murphy'ssign

Chvostek'ssign

Psoassign

Carnett'ssign

Romberg'ssign

Explanation:

Sign Description Typicallyseenin


Abnormalreactionofthefacialnerveto Electrolyteabnormality,most
Chvostek'ssign
stimulation commonlyhypocalcemia
Abdominalwallpaindecreaseswhenthe
abdominalwallmusculatureistensed;
Rectussheathhematoma
Carnett'ssign typicallyindicatingthatthesourceofthe
abdominalwalltrauma
painistheabdominalwall,asopposedto
theabdominalcavity.
Painandtendernesstopalpationofthe
RUQduringinspirationandresultingin
Acutecholecystitis
Murphy'ssign cessationofinspiration;canbeassociated
Liverpathology
withphysicalexaminationor
ultrasonography
Rightlowerquadrantpainwithpassive(or
active)extensionoftherightlower Appendicitis(typically
extremity.Thistypicallyindicatesa retrocecal)
Psoassign
processthatisirritatingtherightpsoas Psoasmuscleabscessor
muscle.(Note:thepatientisontheirside hematoma
duringthisexamination)
Anyprocessthatcauses
Teststhebody'sabilitytosense
dysfunctioninsensory
proprioception(positioning)andthus
Romberg'ssign perception.Thiscanbe
assessfunctionofthedorsalcolumnsof
metabolic(ETOHintoxication)
thespinalcord.
orneuroanatomicalinetiology.
6)Examinationoftheabdomeninapatientsuspectedofhavingappendicitisbeginswith:

rightheeltap
deeppalpation
inspection
auscultation
lightpalpation

Explanation:

Aswithanexaminationforanypurpose,physicalexamshouldbedonethesamewayineverypatientand
shouldalwaysbeginwithinspection.Theabdomenisexposedandthoroughlyinspectedforevidenceofold
surgicalscars,distention,symmetry,masses,visibleperistalsis,hernias,andpulsations,anyofwhichmay
beassociatedwithanacuteabdomen.Inspectionisfollowedbyauscultation,thenlightpalpation,deep
palpationandexaminationforspecialsigns.(Notethatpercussion,whileusefulforageneralorientationto
anontenderabdomen,willbepainfulforthepatientwithperitonitisandshouldnotberoutinelyperformed
beforelightpalpation.)

7) The most common cause of a symmetrically enlarged uterus is


intrauterine pregnancy. Fibroids can also be associated with uterine
enlargement.

Ovarian cysts and tumors may be detected as adnexal masses on one


or both sides, usually non-tender. Cysts tend to be smooth and
compressible, tumors more solid and often nodular. A tender unilateral
adnexal mass in a patient with a positive pregnancy test is an ectopic
pregnancy until proven otherwise.
Acute pelvic inflammatory disease is associated with very tender
bilateral adnexa and purulent cervical discharge; movement of the
uterine cervix produces severe pain. Note that severe pelvic peritonitis
of any etiology can also be associated with cervical motion tenderness.

7) Whichofthefollowinglaboratoryresultswouldbemosttypicalformesentericadenitis?

Elevatedlymphocytecount

ElevatedHematocrit

Elevatedplateletcount

Elevatedneutrophilcount

ElevatedEosinophilcount

Explanation:

Mesentericadenitisbydefinitionisenlargementofthemesentericlymphnodesandthereforeisassociated
withanincreaseintherelativeproportionoflymphocytes.

8)Apreviouslyhealthy12yearoldgirlpresentswithasixhourhistoryofadull,aching,periumbilical
pain,associatedwithalossofappetite.Shevomitedonceanhourago.Vitalsignsincludeabloodpressure
of110/70mmHg,apulseof102perminute,respirationsof18perminute,andatemperatureof100.8F.
Youareconcernedthepatienthasearlyappendicitisandorderlaboratorystudies.Whichlaboratory
findingswouldbeconsistentwiththediagnosisofearlyacuteappendicitis?

>20WBCsperHPFinUrine

Serumtotalbilirubinof7mg/dL

Elevatedserumlipase

12KWhiteBloodCells

>20RBCsperHPFinUrine

Explanation:

Laboratoryresultsneedtobeinterpretedinlightofthepatient'sclinicalpicture.Whilelater/complicated
appendicitiscanhaveawidespectrumofpresentations,earlyappendicitisistypicallyseenwithamildly
elevatedWBC.WhileafewWBCsand/orRBCscanbeseenintheurineinassociationwithappendicitis,
especiallyiftheappendixisinthepelvis,thesetendtonumberlessthan5cellsperHPF.Elevatedlipaseor
bilirubinisnottypicallyseeninearlyappendicitis.

9)An18yearoldgirlpresentswitha12hourhistoryoflowerabdominalpain,whichhaslocalizedtothe
rightlowerquadrant.Shehasnothadanythingtoeatordrinkforthelast12hours.Herlastmenstrual
periodwas2weeksago.Sheisnotsexuallyactive.Onphysicalexaminationtheabdomenistenderinthe
lowerquadrants,withthegreatesttendernesstodeeppalpationintherightlowerquadrant.Pelvic
examinationdemonstratesnocervicalmotiontenderness.Nomassesareappreciated.Vitalsignsincludea
bloodpressureof120/80mmHg,apulseof110perminute,respirationsof18perminute,anda
temperatureof38.0C.Whichofthefollowinglaboratorystudieswouldbemosttypicallyassociatedwith
thishistory?

3+glucoseonurinalysis

3040RBCperhighpowerfieldonurinalysis

WBC13.1

BUNof40

Sodiumof129

Hematocritof25

Explanation:

Labresultsneedtobeinterpretedinlightofthepatient'sclinicalpicture.Whilelater/complicated
appendicitiscanhaveawidespectrumofpresentations,earlyappendicitisistypicallyseenwithamildly
elevatedWBC.WhileafewRBCscanbeseenintheurineinthepresentationofappendicitis,speciallyif
theappendixisinthepelvis,thesetendtobelessthen5cellsperHPF.Anelevatedglucoseintheurineor
anelevatedBUNarenottypicallyseeninearlyappendicitis.Hyponatremiaisnotseeninearlyappendicitis
either.

10)A20yearoldmanpresentswitha24hourhistorylowerabdominalpain,whichhaslocalizedtothe
rightlowerquadrant.Thepatienthasbeennauseated,andhashadlittletoeatordrink.Onphysical
examinationtheabdomenistenderintherightlowerquadrantwithreboundtenderness.Vitalsignsinclude
abloodpressureof120/80mmHg,apulseof105perminute,respirationsof18perminute,anda
temperatureof37.9C.Whichofthefollowingfindingsonurinalysiswouldbemostconsistentwiththis
patient'shistory?

1+protein

3+glucose

20RBCsperhighpowerfieldonurinalysis

whitecellcasts

specificgravity1.026

Explanation:

Whilea"pelvicappendicitis"(aninflamedappendixthathappenstobelocatedinthepelvis)maycause
presenceofWBCsinaurinalysis,theurinalysisistypicallynormalinpatientspresentingwithappendicitis.
Thispatient'spresentationdoesprovideanyreasonforthepresenceofglucose,blood,casts,orproteinin
theurine.Anelevatedspecificgravitycanbeassociatedwithnauseaanddehydration.
11)A45yearoldmanisbroughttotheEmergencyDepartmentbyhiswifebecauseofabdominalpain
andfever.Heunderwentlaparoscopicappendectomyfourdaysagoforaperforatedappendicitis.He
appearsrelativelyhealthyandhaseatenbreakfast;hisbodyhabitusisslightlyobese.Vitalsignsincludea
bloodpressureof135/83mmHg,apulseof102perminute,respirationsof18perminute,anda
temperatureof38.2C.Onphysicalexaminationtherearewellhealingportsites.Theabdomenappears
nondistended.Thereismilddiffuseabdominaldiscomforttopalpation.Bowelsoundsareabsent.The
hemoglobinis13.5.TheWBCis15,000,withanincreaseinthewhitebloodcellcount.Whatisthemost
appropriatenextstepinmanagement?


Outpatientfollowupwithintwodays


AbdominalUltrasound


Diagnosticlaparoscopy


CTscan

Oralantibiotics

Explanation:

WithanelevatedWBCandfeveraccompanyingabdominaldiscomfortonpostopday#4,thereshouldbe
strongsuspicionforacomplicationsuchasanabscesswhichrequiresimmediateevaluation.Thereisno
evidenceoflocalwoundinfection,thereforeantibioticsaloneareinsufficient.Ifanintraabdominalabscess
exists,itcanbemanagedwithpercutaneousdrainage,ratherthanareturntotheOR.CTscanhasamuch
highersensitivityfordetectingabscessthananultrasound,particularlysincetheremaybeanelementof
ileusthatcannegativelyimpactthequalityoftheultrasound.

12)Apreviouslyhealthy42yearoldwomanpresentswitha12hourhistoryofincreasingabdominalpain,
mostmarkedintherightlowerquadrant.Onexaminationtheabdomenisdiffuselytenderwithlocalized
tendernessandreboundintherightlowerquadrant.Vitalsignsincludeabloodpressureof135/83mmHg,
apulseof102perminute,respirationsof18perminute,andatemperatureof38.2C.Laboratorystudies
includeahemoglobinof12.3;theWBCis12,500,withaleftshift.Whichofthefollowingisthemost
appropriateimagingstudybasedonthepatient'shistory?

Abdominalultrasound

CTscanoftheabdomenandpelviswithcontrast

MRI

Technetiumsulfurcolloidscan

Flatanduprightfilmsofthechestandabdomen

Noimagingstudiesareindicated

Explanation:

Imagingstudiesshouldbeconsideredonlywhentheywillaidinnarrowingthedifferentialdiagnosisand
thuschangethepatient'smanagement.Whileappendicitisremainshighonthedifferentialinthispatient,a
12hourhistoryofincreasinganddiffuseabdominalpaininafemale(i.e.relativelyearlytoconsider
perforation)placesovariancyst/rupturehigheronthelist.Thispatientrequiresadetailedgynecological
examination.Thebestimaging,inthiscase,wouldbeapelvicultrasoundexamination.

13)WhichofthefollowingisthemostusefullandmarktofindtheappendixonCTscan?

cecum

rightureter

terminalileum

rightanteriorsuperioriliacspine

rightpsoasmuscle

Explanation:

Whiletheappendixisanatomicallyrelatedtoallstructureslisted,thisrelationshipcanbequietvariable.
Therelationshipoftheappendixtothececumismostconsistentduetotheiranatomicalcontinuity.

14)Apatientpresentswitha24hourhistoryofabdominalpainwhichhasnowlocalizedtotherightlower
quadrant.Onphysicalexamination,thereistendernessofdeeppalpationintherightlowerquadrant,
withoutguardingorrebound.Vitalsignsincludeabloodpressureof135/85mmHg,apulseof100per
minute,respirationsof20perminute,andatemperatureof38.1C.ACTscanisobtained.Basedonthe
CTscanwhatisthemostlikelydiagnosis?


Perforatedappendicitis


Epiploicappendagitis


Appendicealabscess


Acuteappendicitis


Phlegmon

INCORRECT

Explanation:

TheCTshowsafluidfilledtubularstructureintherightlowerquadrantconsistentwiththeappendix.Itis
distendedandlackscontrast,whenthereiscontrastinthececum,suggestingobstruction.Thereisalso
evidenceofmildinflammationintheadjacentfatplanes.Thereisnoevidenceofairorfluidtosuggest
perforationorabscess;orsevereinflammationtosuggestaphlegmon.
15)A65yearoldwomanwithahistoryofmildhypertension,presentswithaoneweekhistoryoffever,
anorexia,abdominalpainandevidenceofdehydration.Vitalsignsincludeabloodpressureof140/85
mmHg,apulseof90perminute,respirationsof22perminute,andatemperatureof101.2F.On
examinationherabdomenissoft.Thereislocalizedmildtendernessandvoluntaryguardingintheright
lowerquadrantwithassociatedfullnessandatenderpalpablemass.Themostlikelydiagnosisis?

Pelvicabscess

Sigmoiddiverticulitis

Cecaldiverticulitis

Periappendicealabscess

Tuboovarianabscess

Meckel'sdiverticulitis

Explanation:

Thepatient'sclinicalpictureismostconsistentwithanintraabdominalinfectiousprocess.Thedifferential
diagnosisina65yearoldwomenincludes,mostcommonly,complicatedappendicitisanddiverticulitis
(sigmoidmorelikelythancecal).TuboovarianabscessandMeckel'sbeinglesslikelyinthisagegroup.
Whilecomplicateddiverticulitisremainsinthedifferentialdiagnosis,therightlowerquadrantlocationand
therelativestatisticalfrequencyofappendicitismakeitthemostlikelydiagnosis.
16)AnabdominalCTscanwithoralandIVcontrastisshown.Themostappropriatenextstepin
managementis:


Percutaneousdrainageoftheabscess

Admissionforhydrationandstabilization

Broadspectrumantibioticsandintervalappendectomy

Evaluationforatumorobstructingtheappendix

Openappendectomy

Explanation:
TheCTscandemonstratesevidenceofawalledoffappendicealabscessthatis5cminmaximaldiameter.
Percutaneousdrainageofaperiappendicealabscessthatisreadilyaccessibleandlargerthan3cmreduces
theneedofmoreextensivesurgeryandallowstheinflammatoryprocesstoresolve,allowingasafer
intervalappendectomyin46weeks.

17)Themostcommonsurgicalcomplicationassociatedwithappendectomyiswhichofthefollowing?

unsightlyscarring

woundinfection

recurrentappendicitis

hemorrhage

inadvertentinjurytothececum
Explanation:

Woundinfectioniscommon,usuallyconfinedtothesubcutaneoustissues,andrespondsfollowingopening
ofthewound.DehiscenceisuncommonwithaMcBurneyincision.Iatrogeniccecalinjurycanoccurwith
eitheropenorlaparoscopicapproachbutisuncommon,asishemorrhage.Recurrentappendicitisisnot
possible;furtherinflammationorfistulashouldraiseconcernforCrohn'sdisease.Scarringisusually
minimalduetothesmallsizeofthesurgicalwounds,evenifleftopentohealbysecondaryintentinthe
faceofgrossperforation.

18)A14yearoldgirlpresentswitha12hourhistoryoflowerabdominalpain,whichhasnowlocalizedto
therightlowerquadrant.Shehasnotedanorexia,butnovomitting.Sheissexuallyactive,butusesbirth
control.Herlastmenstrualperiodwasoneweekago.Onexamination,theabdomenisflat.Bowelsounds
areabsent.Thereisreboundtendernessintherightlowerquadrant.Vitalsignsincludeabloodpressureof
115/70mmHg,apulseof102perminute,respirationsof18perminute,andatemperatureof38.0C.
Laboratorystudiesincludeahemoglobinof12.3,aWBCof13,000withaleftshift.AbetaHCGis
negative.
Whichofthefollowingwouldbearelativecontraindicationtoalaparoscopicappendectomy?

Thepatienthashadmultiplepriorlaparotomies

Thepatienthasaperiappendicealabscess

ThepatienthasCrohn'sdiseaseoftheterminalileuminadditiontoappendicitis

Theappendixisperforated

Smallumbilicalhernia

Noneoftheabove

Explanation:Previousabdominalsurgeriesmayresultinadhesionsthatwouldmakelaparoscopyeither
morechallengingorprohibitive.Sincethereisnoreliablewaytoknowifadhesionswouldexist,savea
historyofsevereadhesionsfoundduringsurgery,thisremainsarelativecounterindication.

19)Thepatientisadvisedtohavelaparoscopicappendectomy.Whichofthefollowingstatements
regardinglaparoscopicappendectomyiscorrect?

Patientsshouldexpecttheplacementofadrainaftersurgery

Itisnotsafeinearlypregnancy

Itislikelytorequirea3dayhospitalization

Woundinfectionisapotentialcomplication

Thepatientshouldnotexpecttoreturntoworkanysoonerthanafteranopenappendectomy

Explanation:Woundinfection,thoughuncommon,isaknownriskoflaparoscopicappendectomy.It
becomesmorelikelyastheappendicitisbecomesmoreadvancedandcomplicatedwithextensiveintra
abdominalinfection.Drainsarenotusedroutinelyfollowinglaparoscopicappendectomy.Hospitalization
shouldgenerallybelessthan36hours.Pregnancyisnotacontraindicationtolaparoscopicappendectomy.
Thepatientshouldexpecttoreturntonormalactivitiessoonerafteralaparoscopicprocedure.
20)Inobtaininginformedconsentforappendectomyinthispatient:

Onlythepatientneedstosigntheconsent

Alternativeoptionsneednotbediscussedasthepatientisaminor.

Thegirl'slegalguardianmustbeinvolvedintheprocessandmustsigntheconsent

Thesurgeonshouldminimizethecomplicationsoftheproceduresoasnottofrightenthepatient

Thecompetencyofthepatientdoesnotneedtobeofconcerniftheprocedureisindicated

Consentisnotrequiredastheprocedureisanemergency

Explanation:Therisks,benefits,andalternativesofeveryproceduremustbeexplainedtoallpatients.If
thepatientisoflegalage,thephysicianobtainingtheconsentmustbeconvincedthatthepatienthas
"decisionalcapacity",meaningtheabilitytounderstandtheparticulardecisionbeingmade.Psychiatric
consultationshouldbesoughtifthereisevidenceofachronicoracutepsychiatricdisorder,whichmay
interferewithdecisionalcapacity.Allphysiciansshouldbeproficientatdeterminingdecisionalcapacity.
Whenatallpossibletheinformedconsentdiscussionmustbedocumentedandconfirmedbysignatureof
thepatientorlegalguardian,healthcareproviderandawitness.Physiciansshouldalwaysactinthebest
interestsofthepatientinanemergency,lifeordeathsituation,whenahealthproxyisnotavailable(e.g.
usuallynextofkinspouse,parentchildinthatorder),andwhentherearenoknownpriordirectivesfrom
thepatient.

21)Apreviouslyhealthy25yearoldmanpresentstotheemergencydepartmentwithprogressiveright
lowerquadrantabdominalpainoverthelast24hours.Hereportsnorecentweightlossandnobloodinhis
stool.
Hisvitalsignsintheemergencydepartmentshow:
Temperature:38.5CHR110RR24BP110/70SaO2100%onroomair
Onphysicalexaminationhehasseveretendernesstopalpationinhisrightlowerquadrant,without
significanttendernessintheremainderofhisabdomen.
LabsareremarkableforaWBCof18,000with15%bandemia
ACTscanofhisabdomenandpelvisshowsathickenedappendixdilatedto1.5cmwithsurroundingfat
stranding.

Whichofthefollowingisthemostlikelyetiologyforhissymptoms?
Periappendicealabscess
Crohnsdisease
Lymphomainvolvingthebaseoftheappendix
Cecalcancerthatincludestheappendicealorifice.

Obstructionoftheappendiceallumenbyapieceofstool
Youransweriscorrect

Thispatienthasacuteappendicitis.Themostlikelyetiologyforthisisobstructionoftheappendiceal
lumenbyafecalithorlymphoidhyperplasia.Lymphomainvolvingthebaseoftheappendixwouldbean
unusualetiologyforappendicitis,andadditionallymphadenopathywouldbeexpectedonphysical
examinationorCT.CrohnsdiseasecanmimickappendicitisbuttheCTwouldmoretypicallyshow
inflammatorychangesinvolvingtheterminalileumoramoreextensiveportionofthececum.Aperi
appendicealabscesswasnotseenontheCT.

22)A28yearoldG2P2femalepresentstotheemergencydepartmentwith12hoursofworsening
abdominalpain.Thepainbeganandhasremainedintherightlowerquadrantofherabdomen.Shereports
nausea,nonbloodydiarrheaand3episodesofnonbiliousemesisduringthistime.
Heryoungestchildis2yearsold.
Onphysicalexamination:
Temperature:38.8CHR115RR:20BP85/60SaO2100%onroomair
Abdomen:Soft,tenderinrightlowerquadrantwithvoluntaryguarding.Theremainderofherabdomenis
nontenderandnotdistended

Whichofthefollowingelementsfromherhistorywillbemostusefulinestablishingthediagnosis?
Familyhistoryofinflammatoryboweldisease
Recenthistoryoftravel
Symptomsofburningwithurination
Historyoftreatedsexuallytransmittedinfection
Recentlystoppedoralcontraceptivepillstoattemptconception
Youranswerisnotcorrect

Arupturedectopicpregnancyisasurgicalemergencyandcanbedifficulttodistinguishfromacute
appendicitis.Althoughinflammatoryboweldiseasecanmimicappendicitis,apositivefamilyhistoryinthe
absenceofothersymptomstosupportthisdiagnosiswouldmakeitlesslikely.Urinarytractinfection
wouldcertainlyneedtobeexcludedwithurinalysisifdysuriaispresent,butdysuriacanalsopresentasa
symptomofappendicitisduetosecondaryinflammationofthebladder.Ahistoryofatreatedsexually
transmittedinfectionmaybeusefulinestablishingthediagnosis,butonlyaftertheabove,moreurgent
diagnoseshavebeenexcluded.

23)A38yearoldmandevelopedsevere,rightlowerquadrantabdominalpain6hourspriortohis
emergencydepartmentpresentation.Priortothis,hewasfeelingwell.
Hehaspoorappetite,nausea,vomiting,diarrhea,andreportsasubjectivefeverathome,thoughhedidnot
checkhistemperature.
Hedeniesdysuria,melena,hematochezia,orahistoryofsimilarsymptoms.Yesterday,hereportshefelt
perfectlywell.
Hehasnosignificantpastmedical,surgicalorfamilyhistory.Hetakesnomedicationsorsupplements
dailyandhasnoknowndrugallergies.

Onphysicalexamination,whichofthefollowingwouldmostsupportyourclinicaldiagnosis:

Observingthatthepatientlieswithhislegsextendedandresistsflexion
Rightupperquadranttenderness,worsewithinspiration
Normalbowelsounds
FeelingHungry

Reproductionofhisrightlowerquadrantabdominalpainwithinternalrotationofhisflexedrighthip
Youransweriscorrect

Flexionandinternalrotationoftherighthipwillputtheobtruatorinternusmuscleonstretch.Ifan
inflamedappendixisincontactwiththismuscle,rightlowerquadrantabdominalpainwillbereproduced.
Thisisobtruatorsignandwouldsupportthedxofappendicitis.PalpationintheRightupperquadrantthat
exhibitstendernessistypicallyassociatedwithGallbladdderdisease.Patientswithinflammationofthe
peritoneumtendtoprefertheirlegsflexedtoreducestretchontheperitoneum.Thispositionalsorelaxes
thepsoasmusclewhichmaybesecondarilyinflamedfromappendicitis.Normalbowelsoundsmayormay
notoccurinapatientwithappendicitis.Thepresenceofobtruatorsignwouldbemuchmorehelpfulto
supportyourclinicaldiagnosis.

24)A19yearoldmanpresentstotheemergencydepartmentwithprogressiverightlowerquadrant
abdominalpainoverthelast24hours,beginningwhilehelpingafriendmoveheavyfurniture.Hereports
nausea,anorexiaandfeverto101.2Fathome.Hehasnothadabowelmovementsincethesymptoms
began.

Whichofthefollowingcomponentsofthephysicalexamwillbemostusefulinestablishingthediagnosis:

Palpationoftheinguinalcanalthroughthescrotum
Digitalrectalexamination
Auscultationoftheabdomen
Askingthepatienttosharplyinhalewhilepalpatingundertheliveredge
Inspectionoftheabdomen
Youransweriscorrect

Itwillbeimportanttoevaluatethepatientforanincarceratedandstrangulatedinguinalherniaasan
etiologyforhisfeverandrightlowerquadrantabdominalpain,whichcanmosteffectivelybeperformedby
palpationoftheinguinalcanalthroughthescrotum.Observationmayalsodemonstrateabulgeor
erythematousskinchangesinthisregion.Auscultationoftheabdomenisunlikelytoprovidemore
informationthandirectpalpation.Askingthepatienttoinhalesharplywhilepalpatingundertheinferior
edgeofthelivertoelicitMurphyssignwouldbemoreimportantifthepatientcomplainedofrightupper
quadrantabdominalpain,orhadsymptomsofcholecystitis.Digitalrectalexaminationmaybehelpfulto
evaluateforfrankoroccultblood,ortenderness,butisunlikelytoprovidemoreusefulinformationthan
evaluationoftheinguinalcanalinthispatient.

25)A65yearoldmalepresentstotheemergencyroomwith12hoursofrightlowerquadrantpain,
leukocytosis,andanorexia.Hispainbeganasperiumbilical,vaguepainandprogressedoverthelast8
hourstofocalpainintherightlowerquadrant.Itisduringthistimeperiodthathelosthisappetite.He
doesnotfeelill.Hasnofever.Onphysicalexam,hehaspointtendernessandguardingtwothirdsofthe
distancefromhisumbilicustohisrightanteriorsuperioriliacspine.Basedonthishistoryandphysical
exam,heisdiagnosedwithtypicalacuteappendicitisandistakentosurgeryforlaparoscopic
appendectomy.Thesurgeondescribestheappendixasnormalappearing.Appendectomyisperformed
withoutcomplication.Finalpathologicalanalysisofhissurgicalspecimenshowsanormalappendix.The
rateofmisdiagnosis,ornegativeappendectomy,inthissituationisbestdescribedas:

65%
15%
35%
50%

5%
Youranswerisnotcorrect

Patientspresentingwithahistory,laboratoryvalues,andexamtypicalofacuteappendicitismaybetaken
tosurgerywithoutimaging.Thesepatientsaremostoftenmalesinthesecondthroughforthdecadeoflife
withawhitecellcount1018K.Alvarodoscoremaybeusedtoevaluatetypicalhistoryforappendicitis.
Despiteimagingtechniquesemployed,misdiagnosiswilloccur.Femalesandthoseage65andolderhave
increasedratesofnegativeappendectomy.

26)A25yearoldmaleisseenintheemergencyroomwithfocalrightlowerquadrantpain.Hehadmild
abdominaldiscomfortlastnightwhichhehasdifficultyspecificallydescribing.Thismorning,hehad
severerightlowerquadrantpainswhichhesaysaremuchworse.Hehasnofever.HisWBCis14kwith
90%PMNs.CTscanoftheabdomenandpelvisidentifiesaninflamedappendixwithanappendicolith.
Heisevaluatedbyasurgeonwhoconfirmsthediagnosisofappendicitisandheistakentosurgeryfor
appendectomy.Heisfoundtohavetypical,nonperforatedappendicitis.Whichofthefollowbest
describestheappropriateuseofantibioticsinthispatient?
Antibioticsshouldbegivenorally
Antibioticsshouldbecontinuedpostoperativelyuntilthepatient'sWBCreturnstonormal
Asingledoseofantibioticpreoperativelyisnecessary
Antimicrobialsarenotnecessaryfornonperforatedappendicitis

Antibioticsshouldbegivenpreoperativelyandfornomorethan24hourspostoperatively
Youranswerisnotcorrect
Severalstudieshavedemonstratedtheefficacyofperioperativeantibioticsinloweringtheinfectious
complicationsofappendicitis.Preoperativeantibioticsareuniversallyaccepted.Patientswithacute
appendicitisshouldhaveantimicrobialscontinuedfornotmorethan24hoursaftersurgery.Ifperforated
organgrenousappendicitisisfound,antibioticsarecontinuedforalongercoursetypicallyuntilsystemic
signsofinfection(eg,WBCelevation,fever)areresolved.

27)Apatientarrivestotheemergencyroomcomplainingofrightlowerquadrantpainwhichwaspresent
whentheyawokethismorningandhasremainedconstant.Noothersymptomsarereported.TheWBC
countis16kwith89%PMNs.Temperatureis37.9C.Allotherlabvaluesarenormal.Abdominalexam
findsfocaltendernessintherightlowerquadrantwithguardingandreboundtenderness.Thistenderness,
althoughsomewhatless,isreproducedwithdeeppalpationandreleaseoftheleftsideoftheabdomen.
Thishistoryandexam,inwhichofthefollowingpatients,warrantsimmediatesurgeryandnofurther
testing.
A31yearoldmalewhotellsyouvomitedonceearlythismorningandremainsanorexic.
A28yearoldfemalewhohasahistoryofCrohn'senteritis.
A28yearoldmalewhoismorbidlyobeseandvomitedonceearlythismorningandisnowhungry
A25yearoldmalewhoreportsbeinghungrydespitehispain.
A27yearoldfemalewhoismarriedandreportsherLMPwasabouttwoweeksago.
Youransweriscorrect

Thediagnosisofappendicitisshouldbequestionedifthepatientisnotanorexic.Inageappropriate
females,painwithovulation(rupturedgraafianfollicle,socalledmittelschmerz)maymimicappendicitis.
Vomitingisverycommoninappendicitisbuttypicallyceasesafteroneortwoepisodes.Chron'senteritis
maypresentwithsymptomssimilartoacuteappendicitis.

28)A29yearoldfemalepresentstotheemergencydepartmentduringher25thweekofpregnancywith
newonsetoffocalrightlowerquadrantpain.Thepainhasbeenconstantfor8hours.Shevomitedearlier
andhasnoteatentoday.Atemperatureof37.5Cisrecorded.WBCis16k.Ultrasoundshowsfreefluidin
therightlowerquadrant,andanappendixdilatedto15mm.Thisexamalsoshowsanormal,viable,intra
uterinepregnancy.Whichofthefollowingstatementsistrueregardingherdiagnosisandtreatment
options?
Diagnosticlaparoscopyisnotanoptionduetopregnancy
CTscanoftheabdomen/pelvisisnecessaryfordefinitivediagnosis.
Regardlessoftreatment,thereisatleasta25%rateofearlydelivery.

Theleukocytosisinthispatientshouldbeconsiderednormal.
Thisisthesecondmostcommonsurgicalemergencyinpregnancy.
Youranswerisnotcorrect

Acuteappendicitisisthemostcommonsurgicalemergencyinpregnancy.Fetallossafterappendectomyis
4%.Earlydeliveryafterappendectomyis710%.Upto25%ofappendectomiesinpregnancyreveala
normalappendix.Thehighestnegativeappendectomyrateoccursinthesecondtrimester;lowestinthird.
Physiologicleukocytosisinnormalpregnancyisupto16k.Nonionizingradiationtechniquestypically
preferredduringpregnancy(ultrasound,MRI)

29)25yearoldmanpresentswith18hoursofanorexiaandperiumbilicalabdominalpain,nowlocalized
intherightlowerquadrant.Onexamhehasatemperatureof101,HR80,BP124/82and02saturation
98%onroomair.HisWBCis14,therestofhislabsarenormal.HisCTscanshowsinflammationinthe
rightlowerquadrantcannotdifferentiateacuteappendicitisfromsigmoiddiverticulitis.Thenextstepin
hismanagementshouldbe:
a.Orderawatersolubleenematoevaluatefordiverticulitis

b.Proceedurgentlytotheoperatingroomforappendectomy
c.Orderbloodculturesandadmitforobservationandantibiotics
d.Discusscasewithradiologisttoseewhatadditionaltestingwouldbehelpful
e.considersendingIBD(inflammatoryboweldisease)bloodpanel
Youransweriscorrect

Althoughanswercisappropriatefortreatmentofdiverticulitis,thispatienthasaclassicconstellationof
symptomsforappendicitisandheisthewrongagefordiverticulitiswhichusuallyaffectsolderpatients,
thereforeanswerbiscorrect.Additionalradiologicalstudiesarenotnecessary,soa.andd.areincorrectas
well.

30)75yearoldfemalewith4weeksofintermittentpostprandialrightupperquadrantabdominalpainwas
seenbyherprimarycarephysicianwhoorderedaCTscanandsentherhome.Hecallsyouurgently
becausetheCTscanwasreadascholelithiasiswithdilatedappendixsuspiciousforacute
appendicitis.Thenextstepinmanagementshouldbe:

a.Callthepatientandscheduleherforanelectivecolonoscopy
b.Callthepatientandscheduleherforurgentappendectomy
c.Callthepatientandscheduleherforurgentcholecystectomy
d.Callthepatientandtellhertocometotheemergencyroom
e.callthepatientandasktohertoproceedtothelabforIBDtesting
Youranswerisnotcorrect

A)isthecorrectanswer.Thispatientdoesnothaveahistoryconsistentwithappendicitisherhistoryis
thatofbiliarycolicwhichisconfirmedwiththectscanfindingsofcholelithiasis.Althoughtheappendix
isabnormalbyCTscan,sincetheclinicalpictureisnotc/wappendicitis,sheshouldnothaveurgent
appendectomy.Thepatientisnotacutelyillsoshedoesnotneedtogototheemergencyroom.Herbiliary
colicwillbetreatedwithelectivecholecystectomyaftertheincidentalfindingsofdilatedappendixare
workedup.Nonpainfulenlargementoftheappendixintheelderlyshouldraisethesuspicionofcolonic
malignancyandacolonoscopyshouldbeperformedtorulethisout.Ifthisisnegative,adiscussionshould
behadwiththepatientaboutthepossibilitiesofaappendicealcystadenomaorcystadenocarcinomaand
plansforelectiveresectionmadeifsheisanacceptablesurgicalcandidate.Hergallbladdermaybe
removedatthesametime.

31)21yearoldfemalecomesinwith12hoursofpainthatbeganaroundtheumbilicusandmigratedtothe
rightlowerquadrant.Shehasafeverof101,awhitebloodcellcountof13andrightlowerquadrant
peritonitisonexam.UrineHCGandpelvicexamisnegative.Themostappropriatediscussiontohavewith
thepatientnextis:
a.Discusswiththepatientincreasedriskofinfertilityassociatedwithappendectomy

b.Discusswiththepatienttheriskofabscessformationassociatedwithappendectomysurgery
c.DiscusswiththepatientrisksofC.diffcolitiswithantibiotics
d.Discusswiththepatienttheriskofdeathassociatedwithappendectomysurgery
e.Discusstheriskofneedingatemporarycolostomy
Youransweriscorrect
Thispatienthadaclassicsymptomcomplexandphysicalexamforappendicitis.Itisnotnecessaryto
performanyothertests(suchasaCTscan)andtheproperactionistoproceedtotheoperatingroomforan
appendectomy.Herchanceoffertilityproblemsarehigheriftreatmentforappendicitisisdelayed,soAis
incorrect.Althoughshewilllikelyreceiveantibioticsforsurgery,theriskforc.diffinahealthyfemale
withoutpriorhospitalizationorexposureisextremelylowandnotthefocusofthetreatment,thereforeI
wouldnotfocusonitunlessantibioticsweretheonlytreatmentplanned.Althoughanswerd.ispartially
correct,theriskofdeathfora21yearoldfemale,althoughnotzero,isextremelyunlikelywithappendicitis
surgery.Inaddition,theriskofdeathishigherfromperforationofherappendixifsurgeryisnot
performed.Answerb.isthebestanswerbecauseitaddressesoneofthemorecommoncomplications
associatedwithcontaminatedcasesorganspaceandwoundinfections.

32)17yearoldmalehighschoolsprinter,presentstotheemergencyroomwithworseningrightlower
quadrantabdominalpainandnauseaoverthelast24hours.Hehadadocumentedfeverof101at
home.Currently,hisvitalsignsarenormalexceptforaheartrateof105andatemperatureof100.5.Hehas
rightlowerquadrantperitonitisonexamatMcBurney'spoint.Whitebloodcellcountis13,000andCT
scanisindeterminateforappendicitisduetononvisualizationoftheappendix.Thenextstepshouldbe:
a.Dischargethepatientforfollowupwithhisprimarycarewithadiagnosisofgastroenteritis

b.ObtainconsultationfromGastroenterology
c.Askthelabtoperformadifferentialonthewhitebloodcellcount

d.Consentthepatientandmotherforanappendectomy
e.Admitthepatientforobservationandserialexam
Youranswerisnotcorrect

Thisisachallengingcasebecauseimagingdoesnotnecessarilysupporttheworkingdiagnosis.Thehistory
andphysicalexamisconsistentwithappendicitiswhiletheCTscanisnot.Answera.isincorrectbecause
youcannotdischargeapatientwhohasabdominalpainandperitonitisgastroenteritisnevercauses
peritonitis.Answerb.isalsoproblematicsinceitwillleadtodelaytodiagnoseasurgicalproblem.
Althoughanswerc.mayprovidemoreinformation,itdoesnotrepresentadecisionfortreatmentsoitisnot
thebestanswer.Thedecisionbetweend.ande.isbasedonthefactthatthepatienthasfocalperitonitis,isa
17yearoldmalewithaclassichistoryandphysicalexamforappendicitisandshouldundergoan
appendectomy.Therearenoperfectradiographicstudiesforappendicitisespeciallyinthinpatients
(sprinter),theappendixcanbeobscuredbyadjoiningbowelloopsandpaucityofintraabdominal
fat.Finally,inapatientwithperitonitis,itisnecessarytoactquicklyunlessthereisadefinitivediagnosis
andplan.Itisalwayssafertoperformanegativeappendectomyinthissettingthantohaveanappendix
perforatewhilethepatientisbeingobserved.

Breast
1) A 62-year-old Asian woman (gravida 3, para 1) comes to the
physicians office for a well-patient visit. She had a fibroadenoma
removed at age 25. Menarche was at age 14 and menopause at age
52. She used oral contraceptive pills for 5 years prior to giving birth to
her first child at age 32. The patient has no known medical problems
and does not smoke cigarettes or drink alcohol.

Physical examination shows no abnormalities.

Which of the following is associated with the greatest increase in risk


for breast cancer development in this patient?
Asian race
Menopause at age 52
First birth at age 32
Menarche at age 14
History of fibroadenoma
Your answer is correct
Late age of 1st full-term pregnancy after age 30 is a mild risk factor
increasing the relative risk of breast cancer. While all women are at
risk for breast cancer, there are a number of factors that increase a
womans chance for developing breast cancer. There are some
important disparities related to race and breast cancer risk. White
women have the highest incidence of breast cancer, while African
American women suffer the highest mortality. Asian women have a
lower incidence of breast cancer than other races but they are less
likely to follow screening guidelines.1 Reproductive history influences
breast cancer risk. Early menarche (before the age of 12) and late
menopause (after the age of 55) are associated with an increased risk
of breast cancer because of a longer lifetime exposure to estrogen.2
Menarche at age 14 or menopause at age 52 would not be associated
with an increased risk of breast cancer. Previous breast biopsies may
influence a womans risk for breast cancer depending upon the
pathology of the biopsy. Proliferative and atypical histology increase
the risk of subsequent breast cancer.3 Fibroadenomas are the most
common benign breast neoplasm and women with simple
fibroadenomas have no increased risk of breast cancer.
2) A healthy 22-year-old woman comes to the physician for evaluation
of mass in the left breast discovered during breast self
examination. Menarche was at age 12. She has 1 child who was born
when she was 20 years of age. She has no family history of breast or
ovarian cancer. Except for an oral contraceptive, which she has used
for 7 years, she takes no medications.

Physical examination discloses a 1-cm, rubbery, smooth, nontender


mass in the upper outer quadrant of the left breast. There is no skin
retraction or lymphadenopathy.

Which of the following is the most likely diagnosis?


Cancer
Galactocele
Fibroadenoma
Cyst
Abscess

Your answer is correct


The most common breast masses in young women are cysts and
fibroademonas.1 Classically, fibradenomas are mobile, solid masses
with well-defined, lobulated borders. Fibroadenomas are usually
nontender with a rubbery feel by palpation.

Typically, a woman with a breast abscess would present with a tender,


fluctuant mass often with overlying erythema. Ultrasound can help
confirm the diagnosis and the treatment is incision and drainage.

The characteristics of a malignant breast mass include firm or hard


consistency and irregular or ill-defined borders. It is important to note,
however, that the absence of these physical exam findings cannot rule
out that a mass is malignant.

Breast cysts present as smooth-walled, fluid filled masses that may be


tender.

Galactoceles are also known as milk retention cysts and they are cystic
fluid collections caused by an obstructed duct. They are most
commonly found in lactating women.

3)A52yearoldwoman(gravida3,para1)comestothephysicianafterdiscoveringa
massintherightbreastduringbreastselfexamination.Menarchewasatage14.Shegave
birthtoherfirstchildatage25.Hermaternalaunthadbreastcancerdiagnosedatage
75.Thepatientsonlymedicationishormonereplacement,whichshehastakenfor5
yearsforsevereperimenopausalsymptoms.

Physicalexaminationdisclosesa1cm,hard,irregular,nontendermassintheouterlower
quadrantoftherightbreast.Thereisnoskinretractionoraxillarylymphadenopathy.

Whichofthefollowingisthemostlikelydiagnosis?
Fibroadenoma
Abscess
Fibrocysticchanges
Cyst
Cancer
Youransweriscorrect

Theevaluationofanywomanwithabreastcomplaintbeginswithathoroughhistoryand
physicalexamination.Animportantcomponentofthepatientshistoryisdeterminingthe
riskfactorsfordevelopingbreastcancer. Ifclinicalbreastexaminationdemonstratesa
singledominantmass,certaincharacteristicsincreasethelikelihoodofthemassbeing
malignantincludingahardconsistency,irregularborders,skinretractionandimmobility.
Therefore, the most likely diagnosis for a hard, irregular, nontender mass in
postmenopausal woman is a cancer.1 It is important to note, however, that physical
findings alone do not reliably differentiate benign from malignant breast lesions and
thereforeadditionalstepsincludingimagingandpossiblebiopsymustbedonetoexclude
breastcancerinawomanwithapalpablemass.

4)A52yearoldwoman(gravida3,para1)comestothephysicianafterdiscoveringa
massintherightbreastduringbreastselfexamination.Menarchewasatage14.Shegave
birthtoherfirstchildatage25.Hermaternalaunthadbreastcancerdiagnosedatage
75.Thepatientsonlymedicationishormonereplacement,whichshehastakenfor5
yearsforsevereperimenopausalsymptoms.

Physicalexaminationdisclosesa1cm,hard,irregular,mobile,nontendermassinthe
outerlowerquadrantoftherightbreast.Thereisnoskinretractionoraxillary
lymphadenopathy.

Whichofthefollowingisthemostappropriatenextstepinmanagement?
Excisionalbiopsy
Mastectomy
Coreneedlebiopsy
Magneticresonanceimagingofthebreast
Mammography
Youransweriscorrect
Afterathoroughhistoryandphysicalexamination,thenextmostappropriatestepinthe
evaluationofanywomanoverthirtywithabreastcomplaintistoobtainadiagnostic
mammogram.Diagnosticmammographydiffersfromscreeningmammographyinthatit
isperformedinwomanwithabreastcomplaintoranabnormalitydetectedbyphysical
examinationorscreeningmammography.Mammographicfindingsthataresuggestiveof
cancer include masses and microcalcifications. The America College of Radiology
classifiesmammogramsaccordingtotheBreastImagingReportingandDetectionSystem
(BIRADS)1below:

BIRADSCategory Recommendation

0Incomplete Needforfurtherevaluation

1Normal Normalintervalfollowup

2Benign Normalintervalfollowup

3ProbablyBenign Shortintervalfollowuprecommended

4SuspiciousAbnormality Biopsyshouldbeconsidered

5HighlySuggestiveofMalignancy Biopsyshouldbeperformed

6BiopsyProvenMalignancy Appropriateactionshouldbetaken

Anormalmammogram,however,doesnoteliminatetheneedforfurtherevaluationofa
breastmassincludingpossiblebiopsy.Somecancersarenotvisiblemammographically.

Although core needle biopsy would eventually be performed in this patient a


mammogram would be done first to characterize the mass and look for any other
nonpalpableabnormalitiesthatmightalsoneedtobeevaluatedfurther.

Magneticresonanceimagingofthebreastwouldnotbeafirststepintheevaluationof
thispatient.Excisionalbiopsyissometimesrequiredfordiagnosisbutitshouldnotbethe
firststep

Mastectomywouldonlybeperformedasamethodoftreatmentforbreastcancernotfor
establishingadiagnosis.
5)Anotherwisehealthy42yearoldwoman(gravida3,para2)comestothephysician
forevaluationof6monthsofpaininherleftbreast.Shehasnohistoryofpreviousbreast
problems.Menarchewasatage12.Shegavebirthtoherfirstchildatage28.The
patientsonlymedicationisanoralcontraceptivepill,whichshehasusedfor5
years.Thereisnofamilyhistoryofbreastorovariancancer.

Onphysicalexamination,thereismildtendernessintheupperouterquadrantoftheleft
breast,butnomassispalpated.Thereisnonippledischargeorinversion,skinchange,or
lymphadenopathy.Mammogramshowsanonspecificsuspiciousabnormalityintheupper
outerquadrantoftheleftbreast.Stereotacticcoreneedlebiopsyofthemassshows
atypicalductalhyperplasia.

Whichofthefollowingisthebestestimateofthiswomansrelativeriskforbreast
cancer?
8.0
2.0
0.5
10.0
4.0
Youranswerisnotcorrect

Highriskproliferativebreastlesionsincludeatypicalductalhyperplasia,atypicallobular
hyperplasia,atypicalpapillaryproliferation,sclerosingbreastlesionswithatypicaland
lobularcarcinomainsitu.Theselesions,althoughnotdirectlypremalignant,doconvey
anincreasedriskofdevelopingbreastcancer.Atypicalductalhyperplasiaisanatypical
proliferativelesionthatcarriesa4foldincreaserelativeriskforbreastcancer.Lobular
carcinomainsitucarriesan8foldincreasedrelativerisk.Inadditiontotheincreasedrisk
ofbreastcancer,whenatypicalproliferativelesionsareidentifiedoncoreneedlebiopsy
thepossibilityofcoexistingoccultmalignancyispresent.Thisriskrangesfrom~10
30%dependingonthelesionandclinicalcircumstances.Giventhisrisk,excisional
biopsytofurthersampletheareashouldbeconsideredfortheselesions.

6)A50yearoldwomancomestothephysicianafteradiagnosticmammogramwith
ultrasoundshowedasuspiciousmassintheupperouterquadrantoftheleft
breast.Menarchewasatage11.Sheusedoralcontraceptivepillsfor8yearsbefore
givingbirthtoherfirstchildatage32.Hermotherhadbreastcancerdiagnosedatage
55.Thereisnootherfamilyhistoryofcancer.

Onphysicalexamination,thebreastsarenontenderwithbilateralsomewhatirregular,
densetissuebutnodiscretemasses.Thereisnolymphadenopathy.

Whichofthefollowingisthemostappropriatenextstepinmanagement?
Incisionalbiopsy
Stereotacticcoreneedlebiopsy
Mastectomy
Lumpectomy
Excisionalbiopsy
Youransweriscorrect

Thepatienthasamammographicabnormalitythatrequiresbiopsy.Imageguidedcore
biopsy has virtually replaced open surgical biopsy in the management of breast
abnormalities. Since the lesion was seen by ultrasound, it is the preferred imaging
modalityforbiopsy.Microcalcificationsareusuallynotvisualizedbyultrasoundunless
theyareassociatedwithamassandthereforemicrocalcificationsmustbebiopsiedusing
mammographic guidance and a stereotactic table. The patient lies prone on the
stereotactictableandherbreasthangsthroughaholeinthetablewhereamammography
unit localizes the area of concern. Some patients cannot tolerate lying prone on the
stereotactictableandrequireneedlelocalizedbiopsyinanoperatingroom.Inaddition,
somelesionslocatednearthenippleorchestwallmayalsonotbeamenabletoimage
guidedbiopsy.1

Incisionalbiopsyinvolvesremovalofonlypartofalesionormassanditisusually
performedwhenlesionsareverylarge.Excisionalbiopsyimpliesremovaloftheentire
lesionbutitisrequiresatriptotheoperatingroomandanincisiononthepatientsbreast
sotherearecostandcosmeticconcerns.Lumpectomyandmastectomyareperformedin
patientswithatissuediagnosisofcancer.

6)A56yearoldwoman(gravida1,para1)comestothephysicianafterdiscoveringa
massintherightbreastduringbreastselfexamination.Menarchewasatage12.Shegave
birthtoheronlychildatage32.Shehasnofamilyhistoryofbreastorovariancancer.

Onphysicalexamination,a2cm,hard,irregular,fixedmassispalpatedinthelower
outerquadrantoftherightbreast,withslightoverlyingskindimplinganda1.5cmhard
massintherightaxilla.Coreneedlebiopsyofthemassshowsductalcarcinoma;fine
needleaspirationcytologyofaxillarynodesshowsadenocarcinoma.

Whichofthefollowingisthebestnextstepinmanagement?
Modifiedradicalmastectomy
Lumpectomyandsentinelnodebiopsy
Chemotherapy
Radiationtherapy
Endocrinetherapy
Youranswerisnotcorrect

ThispatienthasaT3N1M0 StageIIIBorlocallyadvancedbreastcancerwhichisbest
treated with multimodal therapy and usually chemotherapy is administered first (i.e.
neoadjuvantchemotherapy).1Themassisfixedtothechestwallsoradicalsurgerywould
berequiredifanoperationwereperformedfirst.Administrationofchemotherapyina
neoadjuvant(i.e.beforesurgery)fashionmayshrinkthetumorandmayitmobilein
relationshiptothechestwallandtherebypermitalessradicaloperation.

Whileendocrinetherapycanbeusedtotreatbreastcancer,thetreatmentresponsesare
notasrapidandthereforehormonalneoadjuvanttherapyisusuallylimitedtoERpositive
patientswhocanttoleratechemotherapybecauseofcomorbidities.2

Whileradiationtherapywilleventuallyberequiredinthispatienttolessentheriskof
localrecurrence,itisusuallydeliveredafterchemotherapyandsurgery.Lumpectomyand
sentinelnodebiopsyisnotindicatedinthispatientbecausetheprimarytumorislarge
andinvolvesthechestwallandSLNbiopsyisnotindicatedbecauseanFNAofthe
axillary node was positive for tumor cells. Finally, modified radical mastectomy or
removalofthebreastandaxillarycontents wouldnotbeappropriateinitiallyinthis
settingbecausethetumorisfixedtothechestwall.Modifiedradicalmastectomywould
beappropriateifthetumorrespondsfavorablyafterneoadjuvantchemotherapy.

7)Apreviouslyhealthy48yearoldwoman(gravida3,para2)comestothephysician
afterdiscoveringamassintherightbreastduringbreastselfexamination.Menarchewas
atage12.Shegavebirthtoherfirstchildatage22.Herfatherssisterhadbreastcancer
diagnosedatage35.

Onphysicalexamination,a1cm,hard,irregularmassispalpatedintheouterlower
quadrantoftherightbreast.Thereisnoskinretractionoraxillary
lymphadenopathy.Coreneedlebiopsyisperformed,showinginfiltratingductal
carcinoma,HER2/neunegativeandestrogenandprogesteronereceptorpositive.

Whichofthefollowingisthebestnextstepinmanagement?
Endocrinetherapy
Radiationtherapy
Chemotherapy
Simplemastectomy
Lumpectomyandsentinelnodebiopsy
Youransweriscorrect
The majority of women with early stage breast cancer can be treated with breast
conservation.1Therefore,inthispatient,thenextmostappropriatestepinmanagementof
thebreastislumpectomywhiletheaxillacanbestagedwithSLNbiopsy.Sincethe
patientstumoris estrogenandprogesteronereceptorpositive,thepatientisacandidate
forhormonaltherapyaftersurgicaltreatment.Furthermore,thepatientwillrequirebreast
radiationafterlumpectomytoreducetheriskofrecurrenceinthebreast.Chemotherapy
isnotthenextmostappropriatestepinthispatient.Finally,thepatientisacandidatefor
breastconservationandshedoesnotrequiremastectomy.

8)A58yearoldwoman(gravida3,para3)comestothephysicianafterdiscoveringa
massintherightbreastduringbreastselfexamination.Menarchewasatage12.Shegave
birthtoherfirstchildatage25.Thereisnofamilyhistoryofbreastorovariancancer.

Onphysicalexamination,thereisa2cm,hard,irregularmassintheouterlowerquadrant
oftherightbreast,withnoskinretractionoraxillarylymphadenopathy.Thepatient
choosestoundergosimplemastectomywithsentinelnodebiopsyinsteadofbreast
sparingtherapy.Pathologicevaluationofthetissuespecimenshowsa1.9cminfiltrating
ductalcarcinomawithcleansurgicalmargins.Twosentinelnodesshownoevidenceof
malignancy.ImmunohistochemistrytestingshowsthetumorisHER2/neunegativeand
estrogenandprogesteronereceptornegative.

Whichofthefollowingisthenextbeststepinmanagement?
Endocrinetherapy
Completeaxillarydissection
Radiationtherapy
Magneticresonanceimagingoftheaxilla
Chemotherapy
Youranswerisnotcorrect

Adjuvanttherapyforbreastcanceristreatmentgivenaftersurgerywhenpatientsareat
risk for recurrence. Surgery and radiation therapy are local treatments while
chemotherapy and hormonal/endocrine therapy are systemic therapies that address
potentialmicrometastaticdisease.Theindicationforpostoperativesystemictreatmentis
determinedbytumorsize,histology,nodalstatusandtheoverallmedicalconditionofthe
patient. Certain specific tumor characteristics predict response to specific types of
systemictherapy.Forexample,ERpositivepatientsbenefitfromhormonaltherapywhile
HER2positivepatientsbenefitfromHerceptin(trastuzumab)
whichisamonoclonalantibodyagainsttheHer2/neureceptor.1

ThispatientwouldnotbenefitfromanMRIoftheaxillasincetheaxillawasalready
stagedbySLNbiopsy.Furthermore,sincetheSLNwastumorfreethereisnoindication
foraxillarydissection.ThetumorinthispatientwasalsoERnegativeandthereforethe
patient would not benefit from endocrine therapy. Since the patient underwent
mastectomy and the margins were clear there is no indication for postmastectomy
radiationtherapy.

9)Anotherwisehealthy40yearoldwomanisreferredtoabreastsurgeonbyher
gynecologistbecauseofarightbreastmassshefirstnoticedonselfexamination2weeks
ago.Themasswasnotpresentonphysicalexamination1yearago.Shehastwochildren,
ages18and15years.Shetakesadailymultivitaminandcalciumsupplementbutno
othermedications,andshehasnohistoryofsurgery.Thereisnofamilyhistoryofbreast
cancerinherfamily.

Physicalexaminationisunremarkableexceptfora1.5cmfirm,mobile,nontender,
slightlyirregularbutwelldefinedmassintherightbreastupperouterquadrant.Noskin
changesordimplingarenoted.Thereisnolymphadenopathy.

Mammographyshowsdensetissuebutnosuspiciouslesions.Anultrasoundoftheright
breastdoesnotdemostrateacysticlesion.

Whichofthefollowingisthebestnextstepinmanagement?
Coreneedlebiopsy
Fineneedleaspirationbiopsy
Excisionalbiopsy
Wirelocalizationbiopsy
Incisionalbiopsy
Youranswerisnotcorrect

Thereareseveralmethodsavailabletoclinicianstobiopsyabreastabnormality. 1 Itis
importanttorecognizethatsuspiciouspalpablemassesmustbebiopsiedeveniftheyare
notvisiblebybreastimagingsuchasmammographyorultrasound.Somebreastcancers
arenotvisiblebymammographyorultrasound.Theoptimalbiopsymethodprovidesan
accuratediagnosisintheleastinvasiveway.Inthisway,thesurgeoncandiscussoptions
fortreatmentwiththepatientbeforegoingtotheoperatingroom.Biopsytechniques
dependuponwhetherthelesionispalpable.

Fineneedleaspiration(FNA)biopsyinvolvesaspiratingcellsbyrepetitivepassesofa
21gauge needle through a breast lesion. The procedure is well tolerated but has a
relativelyhighfalsenegativeratesuchthatanegativeFNArequiresanothermethodof
tissueacquisitionsuchascorebiopsy.

Wirelocalizationbiopsyisperformedforlesionsthatarenotpalpablebutdetectedby
breast imaging. For example, excision of suspicious microcalcifications is done by
placingawirenearthecalcificationsusingmammography.Thesurgeonthenusesthe
wireasaguidetoremovetheareaandaradiographisdoneofthetissueremovedwith
thewiretoconfirmremovalofthemicrocalcifications (specimenmammogram).The
procedurerequiresthepatientgototheoperatingroomandanincisiononthebreast,
thereforeitisnottheoptimaldiagnosticprocedure.

Coreneedlebiopsyprovidesacoreofbreasttissueasopposedtothecellsprovidedby
FNA,therefore,corebiopsyoffersamoredefinitivehistologicdiagnosis.Corebiopsy
canbedonewithimageguidancetoconfirmpassageofthecoreneedleintothelesion.
Corebiopsypermitsandaccuratediagnosisandifpositiveforcancer,itallowsasurgeon
todiscusstreatmentoptionsandplanforsurgerybeforegoingtotheoperatingroom.

Excisionalbiopsyorsurgicalremovalofalesionisperformedincertainsituations.For
example,

Whencorebiopsyisnotdiagnosticorshowsalesionthatmaybeassociatedwitha
cancersuchasatypicalhyperplasiaorradialscarandexcisionoftheentirelesionis
indicated.

Incisionalbiopsyisrarelyperformedbutinvolvesremovalofpartofalesionthatistoo
bigtobecompletelyremovedbuttheentirelesion.

10)Anotherwisehealthy40yearoldwomanisreferredtoabreastsurgeonbyher
gynecologistbecauseofarightbreastmassshefirstnoticedonselfexamination2weeks
ago.Themasswasnotpresentonphysicalexamination1yearago.Shehastwochildren,
ages18and15years.Shetakesadailymultivitaminandcalciumsupplementbutno
othermedications,andshehasnohistoryofsurgery.Thereisnofamilyhistoryofbreast
cancerinherfamily.

Physicalexaminationisunremarkableexceptfora1.5cmfirm,mobile,nontender,
slightlyirregularbutwelldefinedmassintherightbreastupperouterquadrant.Noskin
changesordimplingarenoted.Thereisnolymphadenopathy.

Mammographyshowsdensetissuebutnosuspiciouslesions.Anultrasoundoftheright
breastdoesnotdemostrateacysticlesion.Coreneedlebiopsydemonstratesinfiltrating
ductalcarcinoma.

Whichofthefollowingisthebestnextstepinmanagement?
Hormonaltherapy
Chemotherapy
Axillarynodedissection
Lumpectomyandsentinellymphnodebiopsy
Modifiedradicalmastectomy
Youransweriscorrect

The treatment of breast cancer is multimodal involving surgery, radiation and


chemotherapy.Treatmentispredicatedonthestageatdiagnosis.Earlystagediseaseis
usuallytreatedwithsurgeryfirst.Surgicaltherapyisdividedintotreatmentofthebreast
andaxilla.Surgicaltreatmentoptionsforthebreastincludelumpectomyormastectomy
withimmediate,delayedornoreconstruction.Somecontraindicationstolumpectomy
includemultifocaldisease,cancerstoobigtoachievetumorfreemarginsandpatient
inabilitytoreceiveradiationtherapy.1

Mastectomyisreservedforpatientswhoarenotcandidatesforbreastconservingsurgery
ordesiremastectomy.Modifiedradicalmastectomyconsistsoftotalmastectomytogether
withaxillarydissection.Inthepatientpresentedinthisquestion,thestatusoftheaxillary
nodeswouldbeassessedsentinelnodebiopsy.

Sentinellymphnode(SLN)biopsyisbasedonthepremisethatthelymphaticdrainageof
abreastcancercanbemappedtooneorafewlymphnodes.Thehistologicstatusofthe
SLNisreflectiveoftheentireaxillaandtherefore,SLNnegativepatientscanbespared
themorbidityofanaxillarydissection.2 Themorbidityofaxillarydissectionincludes
possiblelymphedema,seroma,nerveinjuryandshoulderdysfunction.

Chemotherapyand/orhormonaltherapywouldnotbethenextstepinmanagementofthis
patient.Thesetherapieswouldbedeliveredinanadjuvantfashiondependinguponthe
pathologiccharacteristicsoftheprimarytumor,thehistologyoftheSLNandtheoverall
condition of the patient. Patients with estrogen receptor (ER) positive breast cancer
shouldreceiveendocrinetherapy.Chemotherapyisgiventopatientsbasedontumorsize
andgrade,metastasistotheSLNandmolecularprofilingofthetumor.3

11)Anotherwisehealthy40yearoldwomanisreferredtoabreastsurgeonbyher
gynecologistbecauseofarightbreastmassshefirstnoticedonselfexamination2weeks
ago.Themasswasnotpresentonphysicalexamination1yearago.Shehastwochildren,
ages18and15years.Shetakesadailymultivitaminandcalciumsupplementbutno
othermedications,andshehasnohistoryofsurgery.Thereisnofamilyhistoryofbreast
cancerinherfamily.

Physicalexaminationisunremarkableexceptfora1.5cmfirm,mobile,nontender,
slightlyirregularbutwelldefinedmassintherightbreastupperouterquadrant.Noskin
changesordimplingarenoted.Thereisnolymphadenopathy.

Mammographyconfirmsthepresenceofthe1.5cmlesionbutshowsnoother
abnormalities.Pathologicexaminationofacoreneedlebiopsyspecimenshowsestrogen
andprogesteronereceptornegative,HER2/neunegativeinfiltratingductalcarcinoma.
Lumpectomyshowedclearsurgicalmargins,andsentinelnodeswerenegativefor
carcinoma.

Whichofthefollowingisthebestnextstepinmanagement?
Hormonaltherapy
Observation
Chemotherapyfollowedbyradiation.
Mastectomy
Axillarynodedissection
Youranswerisnotcorrect

ThenextbeststepsinthemanagementofayoungwomanwithaT1 cN0M0breastcancer
who undergoes breast conservation are chemotherapy and radiation. Although the
patientsSLNwastumorfree,shehasatriplenegativebreastcancerasdefinedbythe
absenceoftheestrogenreceptor(ER),progesteronereceptor(PR),andhumanepidermal
growth factor receptor 2 (HER2). Triple negative breast cancers tend to be more
aggressivesothispatientwouldbenefitfromadjuvantchemotherapy.1

Randomized clinical trials demonstrate equivalent outcomes for women treated with
breastconservation compared to mastectomy.2 A 40yearold woman, treated with
lumpectomyshouldreceiveradiationtherapytothebreasttoreducetheriskoftumor
recurrence.Radiationtherapyhastraditionallybeendeliveredbyexternalbeamtothe
whole breast over several weeks. Whole breast radiation therapy reduces the risk of
recurrence by nearly half compared to lumpectomy alone. In highly select patients,
however,alternativemethodsofdeliveringradiationincludeacceleratedpartialbreast
irradiation and intraoperative radiation.3 Accelerated partial breast irradiation is the
deliveryofradiationtothebreasttissuenearthelumpectomysiteoverashorterperiodof
time.Intraoperativeradiationinvolvesgivingtheentiredoseofradiationintheoperating
roomatthetimeofthelumpectomy.

AxillarylymphnodedissectionisnotindicatedinthispatientbecauseherSLNwas
tumorfree.Observationisalsonotappropriateinthisyoungwomanbecauseherriskof
recurrence is high. Hormonal therapy is not indicated because her tumor was ER
negative.Finally,mastectomyisnotnecessarybecauseherlumpectomyshowedtumor
freesurgicalmargins.

ThyroidNodule

1)A32yearoldmalepresentswitha3cmleftthyroidnodule.Hereportscervicaldysphagia.TSHlevels
arenormal.USrevealsbenigncharacteristicsandnootherthyroidnodules.USguidedFNArevealsa
BethesdaCategoryIIbenignnodule.Whatisthenextmostappropriatestepinmanagement?
Subtotalthyroidectomy

Leftthyroidlobectomyandisthmusectomy
CTscanoftheneck
Refertoendocrinologyforradioactiveiodineablation
Referpatientbacktoprimarycareforroutinefollowup
Youransweriscorrect

Discussion:Thispatientpresentswithasymptomaticthyroidnodulecausingdysphagia.AlthoughhisFNA
revealsalowrisknoduleformalignancy(03%basedonBethesdacriteria),thenoduleisstillcausing
troubleswallowing.Inaeuthyroidsetting,radioactiveablationisnotindicated.CTscanoftheneckwould
notprovideanyadditionalinformationinthedecisionmakingprocess.Intheabsenceofmultinodular
goiter,aunilateralprocedureispreferred.

2)A48yearoldfemaleundergoesevaluationforhoarseness.Herworkuprevealsa4.2cmrightthyroid
nodule.SubsequentUSguidedFNArevealsaBethesdaCategoryVInodule,consistentwithpapillary
carcinoma.Whatisthenextmostappropriatestepinhermanagement?

MRIoftheneck
Nuclearmedicinethyroidscan

Laryngoscopy
Rightthyroidlobectomyandisthmusectomy

Totalthyroidectomy
Youranswerisnotcorrect

Discussion:Inpatientswiththyroidcancerwhopresentwithsignsofinvasionintoadjacentstructures,
laryngoscopyshouldbeperformedtoevaluatethefunctionofthevocalcordspreoperatively.Anuclear
medicinethyroidscanwouldnotchangemanagementatthispointinherevaluation,norwouldMRI.A
totalthyroidectomywouldbethedefinitiveprocedureofchoiceafterlaryngoscopytoevaluatehervocal
cords.Althoughlobectomyandisthmusectomyisanappropriatedefinitiveprocedureforsomethyroid
cancers,thispatient'sageandsizeoftumormakeitaninferioroperationinthissetting.

3)A50yearoldfemaleundergoesatotalthyroidectomyforpapillarythyroidcarcinoma.Tumorsizeis
3cmandtherewerenoinvolvedlymphnodes.Postoperativelysheundergoesradioactiveiodineremnant
ablation.Whichofthefollowingisappropriatelongtermfollowupcareforthyroidcancersurveillance?
AnnualTSHmeasurement
AnnualCTscanofthechest
AnnualI131scan

Annualthyroglobulinmeasurement
AnnualCTscanoftheneck
Youranswerisnotcorrect

Discussion:Thyroglobulinisonlyproducedbythyroidcellsandisanexcellentclinicalindicatorof
recurrence.InthesettingofTSHsuppression,aclimbingthyroglobulinlevelshouldpromptfurther
investigationofapossiblerecurrence.TSHlevelsshouldbemonitoredmorefrequentlythanannuallyto
maintainTSHsuppression.CTscansoftheneckorchestarenotindicatedforroutinesurveillance.I 131
scanningcanbeusefulforfurtherevaluationifarecurrenceissuspected.

Pediatric Hernia

1) A2yearoldboyisevaluatedforarightgroinmassthathismotherfirstnoticedduringadiaperchange
3weeksago.Thechildhadanupperrespiratoryinfectionatthetime,andhehadabadcoughingepisodea
fewminutesbeforeherdiscovery.Therespiratoryinfectionhassinceresolved,butthemasspersists.His
motherreportsthatthemassdisappearswhenshepushesitwithherfingers,butithasbeenlessmobile
duringthepast2days.Thechildisotherwisehealthyandtakesnomedications.

Onphysicalexamination,histemperatureis98.6F(37.0C),pulseis84/minandregular,andrespirations
are22/min.Examinationshowsareducablemassintherightgroinabovetheinguinalligament.The
remainderoftheexaminationisunremarkable.

Whichofthefollowingisthemostlikelydiagnosis?
Noncommunicatinghydrocoele

Directhernia
Indirecthernia
Retractiletestis
Femoralhernia
Youranswerisnotcorrect

Inthisyoungagegroup,themostcommonherniawouldbeanindirecthernia.Adirectherniaorafemoral
herniawouldbeveryatypicalinthisagegroupanddoesnotfitthepresentation.Thefactthatthisbulge
comesandgoeswithincreaseabdominalpressure,arguesagainstanoncommunicatinghydrocele.
2)An8montholdboyisbroughttothepediatricianafterhisparentsnotedswellingontherightsideof
thechildsgroin.Theareaisnotsensitivetotouch,buttheswellingseemstobeintermittent.Thechildis
otherwisehealthyandtakesnomedications.

Onphysicalexamination,histemperatureis98.6F(37.0C),pulseis96/minandregular,andrespirations
are26/min.Palpationoftheinguinalareaelicitsthesilkglovesign.

Whichofthefollowingisthemostlikelyexplanationforthisclinicalfinding?
Noncommunicatinghydrocoele
Absenceofaherniasac
Absentvasdeferens

Patentprocessusvaginalis
Retractiletestis
Youranswerisnotcorrect

Asilkglovesignismostconsistentwithapatentprocessusvaginalis.Anoncommunicatedhydrocele
wouldnotpresentwithanintermittentswelling.AbsenceofaVasorasacwouldnotresultinswellingand
aRetractiletestiswouldlikelypresentasapersistentbulgedependingonthelocationofthetestis.

3)A5montholdboyisbroughttothepediatricianforevaluationofasmallmassontheleftsideofthe
infantsgroin.Hersonhadpreviouslybeeningoodhealthuntilhedevelopedaseverecough4daysago.A
diagnosisofcroupwasmade,andhehasbeenrespondingwelltoalbuteroltreatments.Thepatients
motherreportsthatshefirstfeltthemasswhenchanginghisdiaperafteraparticularlybadcoughing
episode.

Onphysicalexamination,temperatureis98.6F(37.0C),pulseis110/minandregular,andrespirationsare
34/min.Onpalpationofthegroinandinguinalarea,nomassispalpable.

Whichofthefollowingisthebeststrategytoestablishadiagnosis?
Placethetipoftheexaminingfingerintheinguinalcanalandpalpateattheexternalringastheinfant
coughs
Elicitcryingbyholdingbothofthechildslegssohecannotmovethem
Palpatethegroinwhilethechildiscomfortableintheparentslap
Observefor10minutes
Palpatetheexternalringduringaprolongeddeepbreath
Youranswerisnotcorrect

Thehistoryismostconsistentwithanindirecthernia.Thisisoftenhardtoelicitunlessthebabyincreases
his/herabdominalpressure(crys).Atypicaladultexam(placethetipoftheexaminingfingerinthecanal
andpalpateastheinfantcoughs)isnotpracticalinthisagegroup.

4) A3yearoldgirlisevaluatedforalumpinthegrointhathermotherdiscoveredthismorningwhile
bathingher.Shesaysthelumpdoesnotseemtobepainfultothechild,whohasbeeneatingandplaying
normallysincebathtime.Thepatientisotherwisehealthyandreceivesnomedications.

Thepatientisplayfulbutapprehensivetoexamination.Vitalsignsarenormal.Examinationofthegroin
showsamobile,nontender,11cmmassintherightgroin.Norednessorswellingoftheskinisnoted
overthemass.Theexternalgenitaliaandabdomenareotherwisenormal.

Thepatientsmassismostlikelywhichofthefollowing?
Lymphnode
Germcelltumor
Dermoidcyst
Testicle

Ovary
Youranswerisnotcorrect

Apersistentbulge,inafemaleinfant,thatislocatedintheinguinalcanalthatisnotrelatedtoincrease
abdominalpressureandnontenderismostlyanovary.Atenderlumpshouldelicitmoreofaconcernfor
aninflamedlymphnode.Withnormalexternalgenitalia,thereisnoreasontothinkthisbulgemaybea
testicleinanotherwisenormalfemale.

5) A3yearoldgirlisevaluatedforalumpinthegrointhathermotherdiscoveredthismorningwhile
bathingher.Shesaysthelumpdoesnotseemtobepainfultothechild,whohasbeeneatingandplaying
normallysincebathtime.Thepatientisotherwisehealthyandreceivesnomedications.

Thepatientisplayfulbutapprehensivetoexamination.Vitalsignsarenormal.Examinationofthegroin
showsamobile,nontender,11cmmassintherightgroin.Norednessorswellingoftheskinisnoted
overthemass.Theexternalgenitaliaandabdomenareotherwisenormal.

Whichofthefollowingisthebestnextstepinmanagement?

Biopsyofthemass
Forciblereductionoftheherniatedmassbeforeitbecomesincarcerated

Reassurancethatthemassisbenignandwilllikelydisappearonitsown
Electiveherniorrhaphy,instructingmothertoremainalertforsignsofstrangulation
Emergentherniorrhaphy
Youranswerisnotcorrect

Herniasininfants,especiallywhenthereisaconcernofboweloranovaryinthehernia,shouldbe
repaired.Thechanceofstrangulationislow,thusthiscanbedoneelectively,andreplacingtheovaryback
intotheabdomen.

6)An8montholdboyisbroughttothepediatricianafterhisparentsnotedintermittentswellingonthe
rightsideofthechildsgroin.Theareaisnotsensitivetotouch.Thechildisotherwisehealthyandtakesno
medications.

Onphysicalexamination,temperatureis98.6F(37.0C),pulseis88/minandregular,andrespirationsare
24/min.Palpationoftheinguinalareaelicitsthesilkglovesign.
Whichofthefollowingisthemostappropriatesurgicalapproachtomanagethispatientscondition?
Laparoscopicclosureoftheexternalring
Suturedmeshovertheinternalring
Approximationofthetransversalisfasciatotheexternalobliqueaponeurosis

Highligationoftheherniasacattheleveloftheinternalring*
Approximationofthetransversalisfasciatotheinguinalligament
Youransweriscorrect

Thepatient'sageandpresentationismostconsistentwithanindirecthernia.Inthisagegroup,theideal
repairofanindirectherniaishighligationofthesacwithouttheuseofmeshorneedforinguinalcanal
floorreinforcement.

7)A4montholdboyisbroughttothephysicianforfollowupevaluation2weeksafterundergoing
surgicalrepairofanindirectinguinalhernia.Theprocedure,simpledissectionandhighligationwith
removaloftheherniasac,wasuncomplicated,andhispostoperativecoursehasbeenuneventful.

Onphysicalexamination,temperatureis98.6F(37.0C),pulseis108/minandregular,andrespirationsare
36/min.Examinationoftheinguinalareashowsawellhealingsurgicalscarandnoswelling.

Whichofthefollowingisthemostappropriateestimatedlifetimeriskofherniarecurrenceforthispatient?
8%

10%
5%
14%

<1%
Youranswerisnotcorrect

Presenceofanindirectherniaatthisagegroupdoesnotnecessarilymeanthattheinguinalcanalhasany
abnormalities,andthusoncethissacisligated,recurrenceisverylow.

PediatricPyloricStenosis

1)

Skin Cancer

1. A66yearoldmanisreferredforevaluationofpainlessleftaxillaryadenopathy.Thepatientisadairy
farmerwhospendsmostdaysworkingoutside,andhehasahistoryofrecurrentsunburns.Hehas
hypertensionthatiswellcontrolledwithanangiotensinconvertingenzymeinhibitor.

Onphysicalexamination,histemperatureis98.6F(37.0C),pulseis76/min,respirationsare14/min,and
bloodpressureis124/76mmHg.Acompleteskinexaminationrevealssignsofchronicsundamagebutno
discretesuspiciouslesion.Cytologicevaluationofanaxillarylymphnodebiopsyisconsistentwith
metastaticmelanoma.

Laboratorystudiesshowanormalserumlactatedehydrogenaselevel.Aradiologicevaluationofthelesion
showsnoevidenceofaprimarytumorordistantmetastaticdisease.

Whichofthefollowingisbestnextstepinmanagement?

Observationonly
Sentinellymphnodebiopsy

Leftaxillarylymphadenectomy
Systemictemozolomide
Palliativeaxillaryradiation
Youransweriscorrect

Thispatientispresentingwithmetastaticmelanomaintheaxillarywithanunknownprimary(MUP)with
noevidenceofdistantdisease.TheincidenceofMUPwas3.2%.Themaletofemaleratiowas2:1while
theagepeakwasinthe4thand5thdecades.Thereisnogreatconsensusonhowtoapproachptswith
MUP,butgoodresponsehasbeenshowntotreatment.Sincethereisnodistantdisease,thepreferred
treatmentmodalitywouldbetocleartheaxillaandthusidentifytheburdenofdisease(andneedfor
radiation).Theptmaybenefitfromadjuvantchemotherapy.

2. A34yearoldwomanwithafamilyhistoryofmelanomaconsultsherphysicianregardingthe
possibilityofundergoinggenetictestingforthedisease.Shereportsthatsheusessunscreenfaithfully
wheneversheisoutdoorsbutisconcernedaboutherriskforskincancerbecauseshefrequentedtanning
salonswhileshewasincollege.Herphysicianexplainsthatpatientswithafamilyhistoryofmelanoma
whoarefoundtocarrythemelanomasusceptibilitygenemayalsobeatincreasedriskforwhichofthe
following?

Bcelllymphoma
Tumorsoftheupperaerodigestivetract
GISTtumors
Papillarythyroidcancer

Pancreaticcancer
Youransweriscorrect

Inadditiontomelanoma,othercancershavebeenobservedinsomemelanomapronefamilieswiththe
melanomasusceptibilitygene.Severalstudieshaveshownanincreasedriskofpancreaticcanceramong
thesemelanomapronefamilies,althoughthepreciserelationshipbetweenthemelanomasuscetibilitygene
andpancreascancerremainsclear.

3. Apreviouslyhealthy36yearoldwomanisevaluatedforalesiononthetopedgeofherleftearthat
shefirstnoticed6monthsago.Thelesionitchesandsometimesbleedswithscratching.Thepatientreports
usingseveraltypesofoverthecounterbodylotionsandhydrocortisonecreamswithoutrelief.Shehasfair
skinandadmitstounprotectedsunexposureandfrequentingtanningsalons.

Onphysicalexamination,hertemperatureis98.6F(37.0C),pulseis72/min,respirationsare18/min,and
bloodpressureis120/74mmHg.Examinationshowsa1.5cmflat,nonpigmented,waxylesiononthe
helixoftheleftear;obviousexcoriationisapparent.

Whichofthefollowingisthemostappropriatenextstepinmanagement?

Fulldermatologicexaminationandinitiationofantibiotictherapy

Fulldermatologicexaminationfollowedbybiopsy
Fulldermatologicexaminationfollowedbyimmediatewideexcision
Detailedheadandneckexaminationfollowedbybiopsy
Immediatebiopsy
Youransweriscorrect

Thepatientspersonalhistoryandthepresenceofasuspiciouslesionwarrantsafulldermatologicexam.
Thelesionsignsandsymptomsaresuspiciousforabasalcellcarcinomaandabiopsyisneededatthis
time.

4. A65yearoldmancomestotheemergencydepartmentbecauseofvomiting,increasinglysevere
crampingabdominalpain,anddistentionforthepast12hours.Hehadnohistoryofpreviousabdominal
surgery.Twoyearsago,heunderwentsurgicalexcisionofa1.5cmtubularmelanomaonhisback.He
takesnomedications.

Onphysicalexamination,histemperatureis98.6F(37.0C),pulseis74/min,respirationsare16/min,and
bloodpressureis122/70mmHg.Theabdomenisdistendedwithhighpitchedtinklingbowelsounds.
Thereissignificanttendernesstopalpation.

Plainradiographsoftheabdomenshowmultipleairfluidlevelswithseveraldistendedloopsofsmall
bowel.

Whichofthefollowingisthebestnextstepinmanagement?

Analgesiaandobservationonly

Computedtomographyoftheabdomen
Exploratorylaparotomy

Positronemissiontomography
Abdominalultrasound
Youranswerisnotcorrect
The patient is presenting with a picture of small bowel obstruction. In
the absence of a history of abdominal surgery, this is worrisome for
obstruction due to a tumor or an internal hernia. With the patient
recent history of melanoma resection, recurrence of melanoma
(metastatic melanoma) a true possibility and a CT scan would help
delineate the nature of obstruction and improve staging (if indeed this
is due to cancer). This is especially applicable since the patients vital
signs are stable.
5. A54yearoldmanreturnstoyourofficeforfollowup1weekafterbiopsyofasuspiciouspigmented
lesiononhisrightcheek.Pathologicevaluationconfirmedasuperficialspreadingmelanomareachinga
maximumdepthof4.2mm.Withtheexceptionofthelesion,thepatientisinotherwisegoodhealth.

Physicalexaminationshowsnoadditionalabnormaldermatologicornodalbasinfindings.

Whichofthefollowingisthebestnextstepinmanagement?

Computedtomographyoftheabdomenandpelvis
Computedtomographyofthechest
Magneticresonanceimagingofthebrain

Positronemissiontomography
Magneticresonanceimagingoftheheadandneck
Youransweriscorrect

TheThickensofthemelanomathisconcerning.Thisisadeepmelanomawithahighincidenceof
metastasisatpresentation.ThusaPETscanisarecommendedscreeningtoolforproperstaging.

6. Apreviouslyhealthy47yearoldwomanisreferredforevaluationofasuspiciousmolenoticedbyher
primarycarephysicianduringaregularphysicalexamination.Becauseofthemoleslocationontheback
ofthepatientsrightshoulder,sheisunsurewhetherithaschangedinsizeorappearance.Shehasfairskin
andadmitsthatshefrequentlysunbathedwithoutprotectivesunscreenduringherteensandtwenties.

Onphysicalexamination,temperatureis98.6F(37.0C),pulseis74/min,respirationsare16/min,and
bloodpressureis122/70mmHgDermatologicexaminationshowsalesionmeasuring4mmindiameter
withslightlyirregularedges;thecolorofthemolevariesfromlighttanononesidetoblackontheother.A
fullthicknessexcisionalbiopsyisperformed,andpathologicreportconfirmsthepresenceofatypical,
immaturemelanocytesextendingintothedermis;Breslowthicknessis1.5mm.

Whichofthefollowingisthebestnextstepinmanagement?

Sentinellymphnodebiopsy
Magneticresonanceimagingofthebrain

Computedtomographyofthechest
Positronemissiontomgraphy
Serummelanomamarkers
Youranswerisnotcorrect
At this time the patient requires prior staging for her intermediate
thickness melanoma. Although for thin melanoma routine radiologic
staging is not recommended, the risk of metastasis is higher in this
patient and baseline radiologic staging would be helpful.
7. A45yearoldmanisevaluatedbecausehiswifeisconcernedthatamoleonhisleftshoulderblade
hasrecentlychangedincolor.Thepatienthaspaleskinandhasexperiencedmultiplesunburnsduringhis
lifetime.Fouryearsago,hereceivedtreatmentforbasalcellcarcinoma.

Onphysicalexamination,histemperatureis98.6F(37.0C),pulseis68/min,respirationsare16/min,and
bloodpressureis122/70mmHg.Dermatologicexaminationshowsa1cmasymmetricallesionontheleft
scapula.Thelesionisdarklypigmentedwithatleastthreeshadesofcolor.

Whichofthefollowingbiopsytechniquesismostappropriate?

Punchbiopsyinthedarkestpartofthemole

Excisionalfullthicknessbiopsy
Punchbiopsyatthemarginofthemole
Shavebiopsy
Incisionalfullthicknessbiopsy
Youransweriscorrect
Since the lesion is small ( 1cm) and present in a location that allows for
complete removal ( back/trunk), the best biopsy would be an excisional
biopsy. This will allow for complete pathological evaluation of the
entire mole and eliminate the potential for a false negative or a
reading of shallow melanoma due to sampling error.

8. A45yearoldmancomestoyourofficeforfollowup1weekafterbiopsyforasuspiciouslesionon
hisleftscapula.Thebiopsysiteishealingwellwithnosignofcomplications.Thepathologyreporthas
returnedwithfindingsconsistentwithsuperficialspreadingmelanomatoadepthof1.2mm.Twoofthe
marginsofthebiopsyareclear.

Whichofthefollowingisthebestnextstepinmanagement?

LocalExcisionMargin SentinelLymphNodeBiopsy? AxillaryLymphNodeDissection?


A. None Yes No
B. 1cm Yes No
C. 2cm Yes No
D. 1cm No Yes
E. 2cm No Yes

C
B
D
A
Youransweriscorrect
The biopsy is consistent with an intermediate thickness melanoma.
This requires both wide local excision ( 2cm) and assessment of the
nodal basin. Since there are no clinically abnormal nodes on exam, a
Sentinel lymph node biopsy is recommended.

9. A68yearoldwomancomestothephysicianforanewpatientvisit.Sixmonthsago,sheunderwent
successfulsurgicaltreatmentforbasalcellcarcinoma.Shehasnochronichealthconditions,andshetakes
nomedications.

Onphysicalexamination,hertemperatureis98.6F(37.0C),pulseis70/minandregular,respirationsare
14/min,andbloodpressureis140/68mmHg.Theexcisionsiteshowsawellhealedscar,andnonew
lesionsareapparent.Theremainderofthephysicalexamination,includingdermatologicexamination,
showsnoabnormalities.

Whichofthefollowingisthemostappropriatemanagementapproachforthispatient?

Regularannualageappropriatephysicalexamination

Sunawarenesseducationandannualfulldermatologicexam
Annualliverfunctiontesting
Annualchestradiograph
Annualdermatologicexaminationtoevaluateforlocalrecurrence
Youransweriscorrect
Due to her personal history of Basal Cell carcinoma, this patient harbors a life-time risk for development of
skin cancer. Giving her good overall health, continued dermatologic surveillance examination is
recommended.

10. A49yearoldmanreturnsfora5yearfollowupexaminationaftersuccessfultreatmentofa
nodulartypemelanomaonhisback.Thelesionwas4.2mminthickness,andwidelocalexcisionand
sentinellymphnodebiopsywereperformed.Heunderwentsubsequentinvolvedlymphaticbasinexcision
andpostoperativechemotherapy.Hereportsthathenowfeelswellwiththeexceptionofheadaches
occurring2to3timesweekly.Hehasbeenveryvigilantwithskinselfexamination.

Onphysicalexamination,histemperatureis98.6F(37.0C),pulseis72/minandregular,respirationsare
16/min,andbloodpressureis124/68mmHg.Afulldermatologicexaminationshowsnoevidenceofany
localorregionalrecurrenceoranynewlesions.Neurologicexaminationshowsnoabnormalities.

Whichofthefollowingisthebestnextstepinmanagement?

Computedtomographyofthechest
Scheduleafollowupexaminationin2years
Positronemissiontomography

Magneticresonanceimagingofthebrain
Scheduleafollowupexaminationin1year
Youransweriscorrect
Headaches presenting in patients with a history of melanoma need to
be further workup to role out brain metastasis. This is true for all
melanoma cases that harbor a risk of metastasis. In this patient,
having had a deep ( 4.2mm) nodular melanoma with nodular
involvement, the risk of distant metastasis and/or recurrence is high.

Trauma
1.A24yearoldwomanisbroughttoyouremergencyroomaftersufferingafallwhileridingherhorse.
SheisimmobilizedinarigidCcollarandonalongspineboard.Sheisalert,unabletomoveanyofher
extremities.Herrespiratorydriveisweaktoabsent,HR78perminute,BP80/52mmHg,andherO2satis
98%beingventilatedwithBagmaskatabout18breathspermin(100%O2).Theappropriatenextstepin
hermanagementwouldbe?


Immediateneurosurgical/orthopedicconsultforprobableCspineinjury.


RadiographicallyassesstheCSpine


Startphenylephrinedrip.


Transfuse2unitsofOnegativeblood


Oraltrachealintubation,maintaininginlineCspineprecautions.

CORRECT

Explanation:
Hypotensionwithouttachycardiainthepresenceofquadriplegiaisconsistentwithneurogenicshock.
AdditionofinsufficientrespiratorydrivetotheclinicalpictureheightenssuspicionofahighCspine
injury.Insuchaclinicalscenario,withthelossofrespiratorydriveandneurogenicshock,securingthe
airwaytakesprecedenceandadefinitiveairwaymustbeobtained.

2.Youaretreatinga50yearoldmanthatwasinvolvedinafire.Hesuffered25%2nd3rddegreeburns
toanteriorchest,neckandface.Hehassingednasalhairs,andyouidentifystridoronyourprimarysurvey.
Pulseoximetryis88%ona100%nonrebreathermask.Aburncenteris45milesaway.Thenextmost
appropriatesteppriortotransferis?


Oraltrachealintubation


Bronchoscopy


CTscanoftheneck


Applysterilesalinetoallwounds


Cricothyroidotomy

INCORRECT

Explanation:

Burns,inthisscenario,exhibitaneffectonairway,breathing,andcirculation.Singednasalhairs,facial
burns,andstridorallsuggestanairwayinjurythatthreatensaneventuallossofairwayduetoprogressive
edemaandswelling.Hypoxiasuggestsaninhalationinjury.Theextentofsecondandthirddegreeburns
willresultinincreasedevaporativefluidlossandshouldbeconsideredandtreatedinthecirculation
componentoftheprimarysurvey.Ofalltheseinjuries,securinganairway(especiallyinlightofan
impendingtransfer)takesprecedence.
3. A69y/omanisinvolvedinamotorvehicleaccidentandisbroughttoyourEDbyambulance.His
vitalsignsareHR:110perminute,BP:170/90mmHg,respirations35perminute.Onaphysical
examination,hehas10cmscalplacerationextendingacrosstheanteriorforehead,raccooneyes,anda
GCSscoreof5.Thenextmostappropriatestepinthemanagementofthepatient'sairwayis


LaryngialMaskAirway


10litersoxygenbynasalcannula


Cricothyroidotomy


Placementofanoralairway


50%oxygenbyfacemask


Orotrachealintubation

INCORRECT

Explanation:

Severeclosedheadinjury(CHI)andevidenceofheadtraumawithGCS<=8,needstobeaggressively
resuscitatedtoavoidasecondaryinsult,i.e.hypoxiaandhypotension.PatientswithaGCS<=8runahigh
riskofairwaylossandthusrequireadefinitiveairway(Orotrachealintubation).

4. A30y/omanarrivesintheEDafteratireexplodedintohisface.Hehasbloodcomingfromhis
mouthandnose,andhisGCSis5.Attemptstoclearhismouthofbloodareunsuccessful.Thenextmost
appropriatestepinthemanagementofthepatient'sairwaywouldbe:


10litersoxygenbynasalcannula


LaryngialMaskAirway


Cricothyroidotomy


50%oxygenbyfacemask


Endotrachealintubation


Placementofanoralairway

CORRECT

Explanation:

PatientswithaGCS<=8runahighriskofairwaylossandthusrequireadefinitiveairway.Endotracheal
intubationistheprimarydefinitiveairwaysought.Inthepresenceoforalmaxillofacialtrauma,
endotrachealintubationmayprovetobedifficultandthusasurgicalairway,cricothyroidotomy,becomes
themethodofchoiceforobtaininganurgentdefinitiveairway.

5.A16yearoldmanisinvolvedinamotorvehicleaccident.Hewasunrestrainedandwasejectedfromthe
vehicleforadistanceof8feet.HeisbroughttotheEmergencyDepartmentalert,butinobviousrespiratory
distress.VitalsignsincludeHR140perminute,BP60/40mmHg,andrespirationsof34perminute.An
initialassessmentdemonstratestendernesstopalpationovertherightchestandcrepitusovertherightchest
andneck,distendedneckveins,andtrachealdeviation;onauscultationoftherightchest,thereareno
breathsoundsandthereishyperresonancetopercussion.Themostlikelydiagnosisiswhichofthe
following?


Tensionpneumothorax


Flailchest


Pericardialtamponade


Massivehemothorax


Simplepnuemothorax


Tracheobronchialinjury

CORRECT

Explanation:

Chesttraumainthepresenceoftachycardiaandhypotensionshouldprecipitateanimmediateruleoutofan
obstructiveshockphenomenon,i.e.tensionpneumothoraxorpericardialtamponade.Thisbecomeseven
moreurgentinthepresenceofjugularvenousdistention.Theabsenceofbreathsoundsandpresenceof
trachealdeviationsuggeststhattensionpneumothoraxisthemorelikelydiagnosis.

6. A16yearoldmanisinvolvedinamotorvehicleaccident.Hewasunrestrainedandwasejectedfrom
thevehicleforadistanceof8feet.HeisbroughttotheEmergencyDepartmentalert,butinobvious
respiratorydistress.VitalsignsincludeHR140perminute,BP60/40mmHg,andrespirationsof34per
minute.Aninitialassessmentdemonstratestendernesstopalpationovertherightchestandcrepitusover
therightchestandneck,distendedneckveins,andtrachealdeviation;onauscultationoftherightchest,
therearenobreathsoundsandthereishyperresonancetopercussion.Themostappropriatenextstepin
managementofthispatientis?


Orotrachealintubation


Ultrasound(FAST)exam


ChestXray


Emergencythoracotomy


Needledecompressionofrightchest

CORRECT

Explanation:
Chesttraumainthepresenceoftachycardiaandhypotensionshouldprecipitateanimmediateruleoutofan
obstructiveshockcomponent,i.e.tensionpneumothoraxorpericardialtamponade.Thisbecomeseven
moreurgentinthepresenceofjugularvenousdistention.Theabsenceofbreathsoundsandpresenceof
trachealdeviationsuggeststhattensionpneumothoraxisthemorelikelydiagnosis.Insuchaclinical
picture,immediateneedledecompressionoftheeffectedchestcavityisindicated.

7. A45yearoldmansuffersmultiplestabwoundstotherightupperchest.Heisalertandorientedbutin
pain.Afterassuringpatencyofhisairway,youplaceachesttubeintherightchestbasedonclinicalhistory
andthephysicalfindings.800ccofbloodisimmediatelyreturnedviathechesttube.VitalsignsincludeHR
144perminute,BP90/48mmHg,respirations20perminute,O2saturationof95%onfacemask.
Peripheralintravenousaccessisunabletobeplacedduetothehypovolemicstate.Whichofthefollowing
isthebestoptiontoobtainaccessforfluidresuscitation?


Femoralveintriplelumencatheterplacement


Cutdownonthecephalicvein


Cutdownonthegreatsaphenousvein


Continuedattemptstoobtainperipheralaccess


Femoralvein9French(introducer)catheterplacement

INCORRECT

Explanation:

Properintravenousaccessisessentialforrapidresuscitationofhemorrhagicshock.Flowdynamicsdictate
thatthepreferredaccesshasalargediameterandashortlength.Thesequalitieswillmaximizeflowand
allowforafasterandmoreaggressiveresuscitationofthepatient.Forexample,a16gaugeperipheral
venouscatheterwillallowhigherflowandthusismoreappropriatethana30cmlongcentraltriplelumen
catheter.Inpatientswhoareinastateofshock,a9French(introducer)catheteristheaccessofchoiceif
largegaugeperipheralintravenousaccessisunobtainable.

Vous aimerez peut-être aussi